Вы находитесь на странице: 1из 118

B. TECH.

5 SEMESTER
DISCRETE MATHEMATICS
(I.T & Comp. Science Engg.)
Th

SYLLABUS
B.Tech (CSE/IT, Discrete Mathematical Structures)
Unit I
Logic: Propositional equivalence, predicates and quantifiers, Methods of proofs, proof
strategy, sequences and summation, mathematical induction, recursive definitions and
structural induction, program correctness.
Counting: The basics of counting, the pigeonhole principle, permutations and
combinations, recurrence relations, solving recurrence relations, generating functions,
inclusion-exclusion principle, application of inclusion-exclusion.
Unit II
Relations: Relations and their properties, n-array relations and their applications,
representing relations, closure of relations, equivalence of relations, partial orderings.
Graph theory: Introduction to graphs, graph terminology, representing graphs and graph
isomorphism, connectivity, Euler and Hamilton paths, planar graphs, graph coloring,
introduction to trees, application of trees.
Unit III
Group theory: Groups, subgroups, generators and evaluation of powers, cosets and
Lagrange's theorem, permutation groups and Burnside's theorem, isomorphism,
automorphisms, homomorphism and normal subgroups, rings, integral domains and
fields.
Unit IV
Lattice theory: Lattices and algebras systems, principles of duality, basic properties of
algebraic systems defined by lattices, distributive and complimented lattices, Boolean
lattices and Boolean algebras, uniqueness of finite Boolean expressions, prepositional
calculus. Coding theory: Coding of binary information and error detection, decoding and
error correction.
Text Books:
1) K.H. Rosen: Discrete Mathematics and its application, 5th edition, Tata McGraw
Hill.Chapter 1(1.1-1.5), Chapter 3(3.1-3.4,3.6), Chapter 4(4.1-4.3,4.5), Chapter
6(6.1,6.2,6.4-6.6) Chapter 7(7.1-7.6), Chapter 8(8.1-8.5,8.7,8.8)
2. C. L. Liu: Elements of Discrete Mathematics, 2 nd edition, TMH 2000.
Chapter 11(11.1 11.10 except 11.7), Chapter 12(12.1 12.8)
3.B.Kalman: Discrete Mathematical Structure, 3 rd edition, Chapter 11(11.1,11.2)

References:

1. Discrete Mathematical Structures: Tremblay and Manohar, Tata


McGraw Hill
2. Discrete Mathematics: 1st edition by Maggard Thomson
3. Discrete M a t h e m a t i c s : Semyour Lipschutz, Varsha Patil IInd
Edition Schaums Series, TMH
4. Discrete M a t h e m a t i c a l Structures: Kolman, B u s b y
a n d R o s s , Prentice Hall India, Edition 3
5. Discrete Mathematics and its application Mott Kendle
6. Discrete Mathematical Structure : G. Shankar Rao, New Age Publisher.
7. Fundamental Approach to Discrete Mathematics Acharjaya D. P.
Sreekumar, New Age Publisher.

Disclaimer
This document does not claim any originality and cannot be used as a
substitute of prescribed text books. The information presented here is
merely a collection by the committee members for their respective
teaching assignments. Various sources as mentioned references at the
beginning of the document as well as freely available materials from the
internet were constituted for preparing this document. The ownership of
the information lies with respective authors or institutions. Further this
document is not intended to be used for commercial purposes and the
committee members are not accountable for any issues, legal or otherwise,
arising out of this document. The committee members make no
representations or warranties with respect to the accuracy or
completeness of the contents of the document and disclaim any implied
warranties of merchantability or fitness for a particular purpose. The
committee members shall not be liable for any loss or profit or any other
commercial, incidental, consequential or any other damages.

Acknowledgement
The committee members gratefully acknowledge Google, NPTEL and different
reference books for getting help for preparation of this lecture note. The committee
members also want to express their gratitude to the persons those who thinks
knowledge should be free and be accessible and sharable without any restrictions so
that every individual on this world has the same opportunity to explore and become
enlightened by the collective gift of mankind.
This lecture note being first draft so there may be some error. Also detail proofs
and some graphs are omitted; however details discussion has been made in the class.
Thus apart from this lecture note students/readers are strongly recommended following
the mentioned books in the references and above all conferring with the faculty
members for thorough knowledge in the subject.

Justification of Learning the Subject:


What is Discrete Mathematics?

Consider an analog clock (One with hands that continuously rotate and show time in continuous
fashion) and a digital clock (It shows time in discrete fashion). The former one gives the idea of
Continuous Mathematics whereas the later one gives the idea of Discrete Mathematics. Thus,
Continuous Mathematics deals with continuous functions, differential and integral calculus etc.
whereas discrete mathematics deals with mathematical topics in the sense that it analyzes data
whose values are separated (such as integers: Number line has gaps)
Example of continuous math Given a fixed surface area, what are the dimensions of a cylinder
that maximizes volume?
Example of Discrete Math Given a fixed set of characters, and a length, how many different
passwords can you construct? How many edges in graph with n vertices? How many ways to
choose a team of two people from a group of n?

Why do you learn Discrete Mathematics?

This course provides some of the mathematical foundations and skills that you need in your
further study of Information Technology and Computer Science & Engineering. These topics
include: Logic, Counting Methods, Relation and Function, Recurrence Relation and Generating
Function, Introduction to Graph Theory And Group Theory, Lattice Theory and Boolean Algebra
etc.
.

Unit I
PROPOSITIONAL LOGIC AND COUNTING THEORY
OBJECTIVES:

After going through this unit, you will be able to :

Define proposition & logical connectives.

To use the laws of Logic.

Describe the logical equivalence and implications.

Define arguments & valid arguments.

To study predicate and quantifier.

Test the validity of argument using rules of logic.

Give proof by truth tables.

Give proof by mathematical Induction.

Discuss Fundamental principle of counting.

Discuss basic idea about permutation and combination.

Define Pigeon hole principle.

Study recurrence relation and generating function.

INTRODUCTION :
Mathematics is assumed to be an exact science. Every statement in Mathematics
must be precise. Also there cant be Mathematics without proofs and each proof needs
proper reasoning.

Proper reasoning involves logic.

Logic is the science of reasoning.

The dictionary meaning of

The rules of logic give precise meaning to

mathematical statements. These rules are used to distinguish between valid & invalid
mathematical arguments.

In addition to its importance in mathematical reasoning, logic has numerous


applications in computer science to verify the correctness of programs & to prove the
theorems in natural & physical sciences to draw conclusion from experiments, in
social sciences & in our daily lives to solve a multitude of problems.

The area of logic that deals with propositions is called the propositional calculus or
propositional logic. The mathematical approach to logic was first discussed by British
mathematician George Boole; hence the mathematical logic is also called as Boolean
logic.

In this chapter we will discuss a few basic ideas.


PROPOSITION (OR STATEMENT)

A proposition (or a statement) is a declarative sentence that is either true or


false, but not both.
A proposition (or a statement) is a declarative sentence which is either true or
false but not both.
Imperative, exclamatory, interrogative or open sentences are not statements in
logic.
Example 1 : For Example consider, the following sentences.
(i)
VSSUT is at Burla.
(ii)
2+3=5
(iii)
The Sun rises in the east.
(iv)
(v)
(vi)

Do your home work.


What are you doing?
2+4=8

(vii)
(viii)
(ix)
ix)
(x)

5<4
The square of 5 is 15.
x 3 2
May God Bless you!

All of them are propositions except (iv), (v),(ix) & (x) sentences ( i), (ii) are true,
whereas (iii),(iv), (vii) & (viii) are false.
Sentence (iv) is command, hence not a proposition. ( v ) is a question so not a
statement. ( ix) is a declarative sentence but not a statement, since it is true or
false depending on the value of x. (x) is a exclamatory sentence and so it is not
a statement.
Mathematical identities are considered to be statements. Statements whi ch are
imperative, ex cl am at or y, i n t e r r o g a t i v e or open are not statements in logic.

Compound statements:

Many propositions are composites that are, composed of sub propositions and
various connectives discussed subsequently. Such composite propositions are
called compound propositions.
A proposition is said to be primitive if it cannot be broken down into simpler
propositions, that is, if it is not composite.
Example 2 : Consider, for example following sentences.
a. The sun is shining today and it is colder than
yesterday
b. Sita is intelligent and she studies every night.
Also the propositions in Example 1 are primitive propositions.
LOGICALOPERATIONS OR LOGICAL CONNECTIVES :

The phrases or words which combine simple statements are called logical
connectives. There are five types of connectives. Namely, not, and, or,
ifthen, iff etc. The first one is a unitary operator whereas the other four are
binary operators.
In the following table we list some possible connectives, their symbols &
the nature of the compound statement formed by them.
Sr. No.

Connective

Symbol

AND

OR

Disjunction

NOT

If....then

Conditional or
implication

If and only if (iff)

Compound statement

Conjunction
Negation

Biconditional

Now we shall study each of basic logical connectives in details.

Basic Logical Connectives:


Conjunction (AND):

If two statements are combined by the word and to form a compound


proposition (statement) then the resulting proposition is called the conjunction of
two propositions.
Symbolically, if P & Q are two simple statements, then P Q denotes the
conjunction of P and Q and is read as P and Q.

Since, P Q is a proposition it has a truth value and this truth value


depends only on the truth values of P and Q.
Specifically, if P & Q are true then P Q is true; otherwise P Q i s
false.
The truth table for conjunction is as follows.
P

PQ

Example 3:

Let P: In this year monsoon is very good.


Q: The rivers are flooded.
Then, P Q: In this year monsoon is very good and the rivers are flooded.
Disjunction (OR) :

Any two statements can be connected by the word or to form a


compound statement called disjunction.
Symbolically, if P and Q are two simple statements, then P Q denotes
the disjunction of P and Q and read as ' P or Q ' .
The truth value of P Q depends only on the truth values of P and Q.
Specifically if P and Q are false then PQ is false, otherwise P Q i s true.
The truth table for disjunction is as follows.
P

PQ

Example 4:

P: Paris is in France
Q 23 6
then P Q : Paris is in France or 2 + 3 = 6.
Here, P Q is true since P is true & Q is False.
Thus, the disjunction P Q is false only when P and Q are both false.
Negation (NOT)

Given any proposition P, another proposition, called negation of P, can be


formed by modifying it by not. Also by using the phrase It is not the case that
or It is false that before P we will able to find the negation.
Symbolically, P Read as not P denotes the negation of P. the truth value of P
depends on the truth value of P

If P is true then P is false and if P is false then P is true. The truth table for
Negation is as follows:
P

Example 5:

Let P: 3 is a factor of 12.


Then Q = P: 3 is not a factor of 12.
Here P is true & P is false.
Conditional or Implication: (Ifthen)
If two statements are combined by using the logical connective
ifthen then the resulting statement is called a conditional statement.

If P and Q are two statements forming the implication if P then


Q then we denotes this implication P Q .
In the implication P Q,
P is called antecedent or hypothesis
Q is called consequent or conclusion.
The statement P Q is true in all cases except when P is true and Q is
false.
The truth table for implication is as follows.
P

PQ

Since conditional statement play an essential role in mathematical


reasoning a variety of terminology is used to express P Q .
i)
ii)
iii)
iv)
v)
vi)
vii)
viii)
ix)
x)

If P then Q
P implies Q
P only if Q
Q if P
P is sufficient condition for Q
Q when P
Q is necessary for P
Q follows from P
if P, Q
Q unless P

Converse, Inverse and Contra positive of a conditional statement :


We can form some new conditional statements starting with a conditional
statement P Q that occur so often. Namely converse, inverse, contra positive.
Which are as follows:
1. Converse: If P Q is an implication then Q P is called the
converse of P Q .
2. Contra positive : If P Q is an implication then the implication
QP is called its contra positive.

3. Inverse: If P Q is an implication then PQ is called its


inverse.
Example 6:
Let P: You are good in Mathematics.
Q: You are good in Logic.
Then, P Q : If you are good in Mathematics then you are good in Logic.
1) Converse: Q P
If you are good in Logic then you are good in Mathematics.
2) Contra positive: QP
If you are not good in Logic then you are not good in Mathematics.
3) Inverse: PQ
If you are not good in Mathematics then you are not good in Logic.
Biconditional Statement: Let P and Q be propositions. The
biconditional statement P Q is the proposition " P if and only if Q " .
The biconditional statement is true when P and Q have same truth values
and is false otherwise.
Biconditional statements are also called bi-implications. It is also
read as p is necessary and sufficient condition for Q.
The truth table for biconditional statement is as follows.
P

PQ

Example 7 : Let P : Ram can take the flight.


Q : Ram buy a ticket.
Then P Q is the statement.
Ram can take the flight iff Ram buy a ticket.
Precedence of Logical Operators:

We

can

construct

compound

propositions

using

n e g a t i o n operator and the logical operators defined so far.

the

We will

generally use parentheses to specify the order in which logical operators in a


compound proposition are to be applied. In order to avoid an excessive number
of parentheses.

We sometimes adopt an order of precedence for the logical connectives.

The

following table displays the precedence levels of the logical operators.


Operator

Precedence

LOGICAL EQUIVALANCE:

Compound propositions that have the same truth values in all possible
cases are called logically equivalent.
Definition: The compound propositions P and Q are said to be logically
equivalent if P Q is a tautology. The notation P Q denotes that P and Q
are logically equivalent.
Some equivalence statements are useful for deducing other equivalence statements.
The following table shows some important equivalence.
Logical Identities or Laws of Logic:
Name
1. Identity Laws

Equivalence
PT P
PF P

2. Domination Laws

PT T
PF F

3. Double Negation

P P

4. Idempotent Laws

PP P
PP P

5. Commutative Laws

PQ QP
PQ QP

6. Associative Laws

P Q R P Q R
P Q R P Q R

7. Distributive Laws

P Q R P Q P R
P Q R P Q P R

8. De Morgans Laws

P Q PQ
P Q PQ

9. Absorption Laws

P P Q P
P P Q P

10. Negation Laws


(Inverse / Complement)

P P T

11. Equivalence Law

P Q P Q Q P

12. Implication Law

P Q P Q

13. Biconditional Property

P Q P Q P Q

P P F

14. Contra positive o f


C o ndit io nal statement

P Q QP

Note that while taking negation of compound statement every or


All is interchanged by some & there exists is interchanged by at least one &
vice versa.
Example 8: If P: This book is good.
Q: This book is costly.
Write the following statements in symbolic form.
a) This book is good & costly.
b) This book is not good but costly.
c) This book is cheap but good.
d) This book is neither good nor costly.
e) If this book is good then it is costly.
Answers:
a) P Q
b) P Q
c) Q P
d) P Q
e) P Q
Logical Equivalence Involving Implications :

Let P & Q be two statements.

The following table displays some useful equivalences for implications


involving conditional and biconditional statements.

Sr. No.
1
2
3
4

Logical Equivalence involving implications


P Q P Q
P Q Q P
P Q P Q

P Q P Q

P Q P r P Q r
P r Q r P Q r
P Q P r P Q r
P r Q r P Q r
P Q P Q Q P

7
8
9
10

P Q P Q

11
12

P Q P Q

13

P Q P Q

P Q P Q P Q

All these identities can be proved by using truth tables.


NORMAL FORM AND TRUTH TABLES :

Well ordered Formulas:


A compound statement obtained from statement letters by using one or more
connectives is called a statement pattern or statement form. thus, if P, Q, R,
are the statements (which can be treated as variables) then any statement
involving these statements and the logical connectives ,,,,
statement form or a well ordered formula or statement pattern.

is a

Definition: A propositional variable is a symbol representing any proposition.


Note that a propositional variable is not a proposition but can be replaced by a
proposition.
Any statement involving propositional variable and logical connectives is a well
formed formula.
Note: A wof is not a proposition but we substitute the proposition in place of
propositional variable, we get a proposition.
E.g. P Q Q R Q, P Q etc.
Truth table for a Well Formed Formula:
If we replace the propositional variables in a formula by propositions, we get a
proposition involving connectives. If involves n propositional constants, we get
2n possible combination of truth variables of proposition replacing the variables.
Example 9: Obtain truth value for P Q Q P .
Solution: The truth table for the given well formed formula is given below.
P

PQ

QP

Tautology:
A tautology or universally true formula is a well formed formula, whose truth
value is T for all possible assignments of truth values to the propositional
variables.
Example 10 : Consider P P , the truth table is as follows.
P

P P

P P always takes value T for all possible truth value of P, it is a


tautology.

Contradiction or fallacy:
A contradiction or (absurdity) is a well formed formula whose truth value is
false (F) for all possible assignments of truth values to the propositional
variables.
Thus, in short a compound statement that is always false is a contradiction.
Example 11 : Consider the truth table for P P .
P

P P

P P always takes value F for all possible truth values of P, it is a


Contradiction.
Contingency:
A well formed formula which is neither a tautology nor a contradiction is
called a contingency.
Thus, contingency is a statement pattern which is either true or false depending
on the truth values of its component statement.
Example 12: Show that p q and pq are logically equivalent
.
Solution : The truth tables for these compound proposition is as follows.
1

PQ

P Q

P Q

67

We can observe that the truth values of p q and p q agree for all possible
combinations of the truth values of p and q.

It follows that p q p q is a tautology; therefore the


given compound propositions are logically equivalent.
Example 13: Show that p q and p q are logically equivalent.
Solution : The truth tables for these compound proposition as follows.
p
T
T
F
F

p
F
F
T
T

q
T
F
T
F

p q
T
F
T
T

pq
T
F
T
T

As the truth values of p q and p q are logically equivalent.


Example 14 : Determine whether each of the following form is a
tautology or a contradiction or neither :
i) P Q P Q
ii)
iii)
iv)
v)

P Q P Q
P Q P Q
P Q P Q
P P Q Q

Solution:
i) The truth table for p q p q

pq

pq

p q p q

Here all the entries in the last column are T.


p q p q is a tautology.

ii) The truth table for p q p q is


1

pq

P q

36

The entries in the last column are F. Hence


contradiction.

p q p q

is a

iii) The truth table is as follows.


p

p q

pq

p q p q

Here all entries in last column are T.


p q p q is a tautology.

iv) The truth table is as follows.

p q

pq

p q p q

All the entries in the last column are F. Hence it is contradiction.

v)

The truth table for p p q q

p q

p p q

p p q q

The last entries are neither all T nor all F.


p p q q is a neither tautology nor contradiction. It is a
Contingency.

PREDICATES AND QUANTIFIERS


Predicates: A predicate is a function from universe of discourse to truth values.
Consider a sentence: x is greater than 2. Here is greater than 2 is the predicate and x
is the subject or variable.

If values are assigned t o all the variables, t h e resulting sentence is a proposition.


e.g. 1. x < 9 is a predicate
2. 4 < 9 is a proposition
Propositional Function:

A propositional function (or

an open sentence) defined on A is a predicate

together with subjects. It is denoted by the expression P(x) which has the
property that P(a) is true or false for each a A .
The set A is called domain of P(x) and the set Tp of all elements of A for which P (a)
is true is called the truth set of P(x).
Propositional functions can be converted to proposition by two aspects
(i) By assigning exact value to the variable and (ii) using quantification.
e.g. Let A = {x / x is an integer < 8}
Here P(x) is the sentence x is an integer less than 8.
The common property is an integer less than 8.
P(1) is the statement 1 is an integer less than 8.
P(1) is true.

Quantifiers:

Quantification is the way by which a Propositional function can be turns out to


be a proposition. The expressions for all and there exists are called
quantifiers. The process of applying quantifier to a variable is called
quantification of variables.
Universal quantification:
The universal quantification of a predicate P(x) is the statement, For all values
of x, P(x) is true.
The universal quantification of P(x) is denoted by
The symbol
e.g.

for all x P(x).

is called the universal quantifier.

1) The sentence P(x) : - (-x) = x is a predicate that makes sense for real
numbers x. The universal quantification of P(x),

x P(x) is a true

statement because for all real numbers, -(- x) = x.

2) Let Q(x) : x + 1 < 5, then

Q(x) : x + 2 < 5 is a false statement, as

Q(5) is not true. Universal quantification can also be stated in English as


for every x, every x, or for any x.
Existential quantification The existential quantification of a predicate P(x) is the statement
There exists a value of x for which P(x) is true.
The e x i s t e n t i a l q u a n t i f i c a t i o n o f P (x) i s d e n o t e d xP( x) . The
symbol is called the existential quantifier. e.g.
1)

Let Q : x + 1< 4 . The existential quantification of Q(x), xQ( x)


is a true statement, because Q(2) is true statement.

2)

The statement y, y + 2 = y is false. There is no value of y for


which the propositional function y+2 = y produces a true statement.

Negation of Quantified statement :

x p( x )= v x p(x)
and v x p(x)= x p(x)

This is true for any proposition p(x).


For example, The negation of all men are mortal is: There is a man who is not mortal.
Example 15 :
Express the statement using quantifiers: Every student in your school has a
computer or has a friend who has a computer.
Solution :
Let c(x) : x has a computer
F(x,y) : x and y are friends
Thus, We have
v x(c( x) y(c( y) F ( x, y))

THEORY OF INFERENCE FOR THE PREDICAT CALCULAS


If an implication P Q is a tautology where P and Q may be compound statements
i n v o l v i n g any number of propositional variables we say that Q logically follows
from P. Suppose P(P1 ,P2 .......Pn ) Q . Then this implication is true regardless of the
truth values of any of its components. In this case, we say that Q logically
follows from P1, P2..,Pn.
Proofs in mathematics are valid arguments that establish the truth of mathematical
statements.
To deduce new statements from statements we already have, we use rules of
inference

which

are t e m p l a t e s

for c o n s t r u c t i n g

valid

arguments. Rules of inference are our basic tools for establishing the truth of
statements. The rules of inference for statements involving existential and
universal quantifiers play an important role in proofs in Computer Science and
Mathematics, although they are often used without being explicitly mentioned.
Valid Argument:
An argument in propositional logic is a sequence of propositions. All propositions
in the argument are called hypothesis or Premises.The final proposition is called
the conclusion. An argument form in propositional logic is a sequence of
compound propositions - involving propositional variables.
An argument form is valid if no matter which particular propositions are
substituted for the propositional variables in its premises, the conclusion is true if
the premises are all true.

Thus we say the conclusion C can be drawn from a given set of premises or the
argument is valid if the conjunction of all the premises implies the conclusion is
a tautology.
Rules of Inference for Propositional logic
We can always use a truth table to show that an argument form is valid. Arguments
b a s e d o n t a u t o l o g i e s r e p r e s e n t u n i v e r s a l l y c o r r e c t method
o f r e a s o n i n g . Their validity depends only on the form of statements
involved and not on the truth values of the variables they contain such
arguments are called rules of inference.
These rules of inference can be used as building blocks to construct more
complicated valid argument forms
e.g.
Let

P: You have a current password


Q: You can log onto the network.

Then, the argument involving the propositions,


If you have a current password, then you can log onto the network.
You have a current password therefore: You can log onto the network has
the form
.
P Q
P
Q

Where
is the symbol that denotes therefore we know that when P & Q are proposition
variables, the statement ((P Q) P) Q is a tautology
.
So, this is valid argument and hence is a rule of inference, called modus ponens or the
law of detachment.
(Modus ponens is Latin for mode that affirms)
The most important rules of inference for propositional logic are as follows..

Example16:

Test the validity of the following arguments :


1.

If milk is black then every crow is white.

2.

If every crow is white then it has 4 legs.

3.

If every crow has 4 legs then every Buffalo is white and brisk.

4.

The milk is black.

5.

So, every Buffalo is white.

Solution :
Let

P : The milk is black


Q : Every crow is white
R : Every crow has four legs.
S : Every Buffalo is white
T : Every Buffalo is brisk

The given premises are


(i)

PQ

(ii)

QR

(iii)

R ST

(iv)

The conclusion is S. The following steps checks the validity of argument.


1.

PQ

premise (1)

2.

QR

Premise (2)

3.

PR

line 1. and 2. Hypothetical syllogism (H.S.)

4.

R ST

Premise (iii)

5.

P ST

Line 3. and 4.. H.S.

6.

Premise (iv)

7.

ST

Line 5, 6 modus ponens

8.

Line 7, simplification

The argument is valid

Example17 :
Consider the following argument and determine whether it is valid or not. Either I will

get good marks or I will not graduate. If I did not graduate I will go to USA. I get
good marks. Thus, I would not go to USA.
Solution :
Let
P : I will get good marks.

Q : I will graduate.
R : I will go to USA
The given premises are
i)
PVQ
ii)
QR
iii)
P

The conclusion is R.

What are proofs?

A proof is a clear explanation, accepted by the mathematical community, of why


something is true.
Ancient Babylonian and Egyptian mathematics had no proofs, just examples and methods.
Proofs in the way we use them today began with the Greeks and Euclid
Methods of Proof:
There are different methods of proof as follows:
Direct method
Indirect method.
Contradiction method.
Vacuous method.
Method of induction etc
Already you have the idea about above mentioned methods. Let us discuss
method of induction.
MATHEMATICAL INDUCTION
Here we discuss another proof technique. Suppose the statement to be proved
can be put in the from P(n), n n0. where n0 is some fixed integer.

That is suppose we wish to show that P(n) is true for all integers n n0.

The following result shows how this can be done.


Suppose that
(a)

P(n0) is true and

(b)

If P(K) is true for some K n0, then P(K + 1) must also be


true. The P(n) is true for all n n0.

This result is called the principle of Mathematical induction.


Thus to prove the truth of statement

nn0. P(n), using the

principle of mathematical induction, we must begin by proving directly that


the first proposition P(n0) is true. This is called the basis step of the induction
and is generally very easy.
Then we must prove that P(K) P(K + 1) is a tautology for any choice
of K n0. Since, the only case where an implication is false is if the antecedent

is true and the consequent is false; this step is usually done by showing that if
P(K) were true, then P(K + 1) would also have to be true. This step is called
induction step.
In short we solve by following steps.
1.

Show that P(1) is true.

2.

Assume P(k) is true.

3.

Prove that P(k +1) is true using

P(k) Hence P(n) is true for every n.


Example 18 :
Using principle of mathematical induction prove that

(i)

1 + 2 + 3 + ... + n = n (n + 1) / 2

(ii)
(iii)
(iv)
(v)
(vi)

1 2 + 2 2 + 3 2 + ... + n 2 = n (n + 1) (2n + 1)/ 6


1 3 + 2 3 + 3 3 + ... + n 3 = n 2 (n + 1) 2 / 4
3 n > n 2 for n = 1, n = 2
3 n > n 2 for n a positive integer greater than 2.
For any positive integer number n , n 3 + 2 n is divisible by 3

Solution (i)
Let the statement P (n) be
1 + 2 + 3 + ... + n = n (n + 1) / 2
STEP 1: We first show that p (1) is true.
Left Side = 1
Right Side = 1 (1 + 1) / 2 = 1
Both sides of the statement are equal hence p (1) is true.
STEP 2: We now assume that p (k) is true
1 + 2 + 3 + ... + k = k (k + 1) / 2
and show that p (k + 1) is true by adding k + 1 to both sides of the above
statement
1 + 2 + 3 + ... + k + (k + 1) = k (k + 1) / 2 + (k + 1)
= (k + 1)(k / 2 + 1)
= (k + 1)(k + 2) / 2

The last statement may be written as


1 + 2 + 3 + ... + k + (k + 1) = (k + 1)(k + 2) / 2
Which is the statement p(k + 1).
Hence , by method of induction P(n) is true for all n.
Solution (ii)
Statement P (n) is defined by
1 2 + 2 2 + 3 2 + ... + n 2 = n (n + 1) (2n + 1)/ 2
STEP 1: We first show that p (1) is true.
Left Side = 1 2 = 1
Right Side = 1 (1 + 1) (2*1 + 1)/ 6 = 1
Both sides of the statement are equal hence p (1) is true.
STEP 2: We now assume that p (k) is true
1 2 + 2 2 + 3 2 + ... + k 2 = k (k + 1) (2k + 1)/ 6
and show that p (k + 1) is true by adding (k + 1) 2 to both sides of the above
statement
1 2 + 2 2 + 3 2 + ... + k 2 + (k + 1) 2 = k (k + 1) (2k + 1)/ 6 + (k + 1) 2
Set common denominator and factor k + 1 on the right side
= (k + 1) [ k (2k + 1)+ 6 (k + 1) ] /6
Expand k (2k + 1)+ 6 (k + 1)
= (k + 1) [ 2k 2 + 7k + 6 ] /6
Now factor 2k 2 + 7k + 6.
= (k + 1) [ (k + 2) (2k + 3) ] /6
We have started from the statement P(k) and have shown that
1 2 + 2 2 + 3 2 + ... + k 2 + (k + 1) 2 = (k + 1) [ (k + 2) (2k + 3) ] /6
Which is the statement P(k + 1).
Hence , by method of induction P(n) is true for all n.

Solution (iii)
Statement P (n) is defined by
1 3 + 2 3 + 3 3 + ... + n 3 = n 2 (n + 1) 2 / 4
STEP 1: We first show that p (1) is true.
Left Side = 1 3 = 1
Right Side = 1 2 (1 + 1) 2 / 4 = 1
hence p (1) is true.
STEP 2: We now assume that p (k) is true
1 3 + 2 3 + 3 3 + ... + k 3 = k 2 (k + 1) 2 / 4
add (k + 1) 3 to both sides
1 3 + 2 3 + 3 3 + ... + k 3 + (k + 1) 3 = k 2 (k + 1) 2 / 4 + (k + 1) 3
factor (k + 1) 2 on the right side
= (k + 1) 2 [ k 2 / 4 + (k + 1) ]
set to common denominator and group
= (k + 1) 2 [ k 2 + 4 k + 4 ] / 4
= (k + 1) 2 [ (k + 2) 2 ] / 4
We have started from the statement P(k) and have shown that
1 3 + 2 3 + 3 3 + ... + k 3 + (k + 1) 3 = (k + 1) 2 [ (k + 2) 2 ] / 4
Which is the statement P(k + 1).
Hence , by method of induction P(n) is true for all n.
Solution (iv)
Statement P (n) is defined by
n 3 + 2 n is divisible by 3
STEP 1: We first show that p (1) is true. Let n = 1 and calculate n 3 + 2n
1 3 + 2(1) = 3

3 is divisible by 3
hence p (1) is true.
STEP 2: We now assume that p (k) is true
k 3 + 2 k is divisible by 3
is equivalent to
k 3 + 2 k = 3 M , where M is a positive integer.
We now consider the algebraic expression (k + 1) 3 + 2 (k + 1); expand it and group like
terms
(k + 1) 3 + 2 (k + 1) = k 3 + 3 k 2 + 5 k + 3
= [ k 3 + 2 k] + [3 k 2 + 3 k + 3]
= 3 M + 3 [ k2 + k + 1 ] = 3 [ M + k2 + k + 1 ]
Hence (k + 1) 3 + 2 (k + 1) is also divisible by 3 and therefore statement P(k + 1) is true.

Hence , by method of induction P(n) is true for all n.


Solution (v)
Statement P (n) is defined by
3n > n2
STEP 1: We first show that p (1) is true. Let n = 1 and calculate 3 1 and 1 2 and
compare them
31 = 3
12 = 1
3 is greater than 1 and hence p (1) is true.
Let us also show that P(2) is true.
32 = 9
22 = 4
Hence P(2) is also true.
STEP 2: We now assume that p (k) is true
3k > k2

Multiply both sides of the above inequality by 3


3 * 3k > 3 * k2
The left side is equal to 3 k + 1. For k >, 2, we can write
k 2 > 2 k and k 2 > 1
We now combine the above inequalities by adding the left hand sides and the
right hand sides of the two inequalities
2 k2 > 2 k + 1
We now add k 2 to both sides of the above inequality to obtain the inequality
3 k2 > k2 + 2 k + 1
Factor the right side we can write
3 * k 2 > (k + 1) 2
If 3 * 3 k > 3 * k 2 and 3 * k 2 > (k + 1) 2 then
3 * 3 k > (k + 1) 2
Rewrite the left side as 3 k + 1
3 k + 1 > (k + 1) 2
Which proves that P(k + 1) is true
Hence , by method of induction P(n) is true for all n.
Solution (vi)
Statement P (n) is defined by
n! > 2 n
STEP 1: We first show that p (4) is true. Let n = 4 and calculate 4 ! and 2 n and compare
them
4! = 24
2 4 = 16
24 is greater than 16 and hence p (4) is true.
STEP 2: We now assume that p (k) is true

k! > 2 k

Multiply both sides of the above inequality by k + 1


k! (k + 1)> 2 k (k + 1)
The left side is equal to (k + 1)!. For k >, 4, we can write
k+1>2
Multiply both sides of the above inequality by 2 k to obtain
2 k (k + 1) > 2 * 2 k
The above inequality may be written
2 k (k + 1) > 2 k + 1
We have proved that (k + 1)! > 2 k (k + 1) and 2 k (k + 1) > 2 k + 1 we can now
write
(k + 1)! > 2 k + 1
We have assumed that statement P(k) is true and proved that statement P(k+1) is
also true.
Hence , by method of induction P(n) is true for all n.

COUNTING:

Broadly speaking combinatory(counting) is the branch of mathematics dealing


with order and patterns without regard to the intrinsic properties of the objects under
consideration.
FUNDAMENTAL PRINCIPLE COUNTING (FPC):

The two main counting rules: The Multiplication Rule states that if one can do a
job by doing two tasks one after the other, and there are m ways to do the first task and
then n ways to do the second, then there are mn ways to do the whole job.
For Example, suppose there are 3 routes from Burla to Sambalpur and 4 routes from
Sambalpur to Cuttack, then by FPC the total number of ways for performing journey
from Burla to Cuttack is 12.
The Addition Rule, states that if one can do a job by doing one or the other (but not
both) of two tasks, and there are m ways to do then first task and n ways to do the
second, then there are m+n ways to do the whole job.
PERMUTATIONS AND COMBINATIONS:

Permutation is the arrangement of objects with ordering, whereas combination is the


selection of objects without ordering.
Permutation Formula:

The permutation of n things taken r at a time without repetition is

(i)

P(n, r) = n!/(n - r)!


n = the total number of items you have from which to take
r = the number you are actually going to use.
(ii)
(iii)

The permutation of n things taken r at a time with repetition is


P(n, r) = n r
The permutation of n things taken all at a time with repetition is
P(n,n) = n!

Factorial Rule: For n items, there are n! (pronounced n factorial) ways to arrange them.

n! = (n)(n - 1)(n - 2). . . (3)(2)(1)


For example:
3! = (3)(2)(1) = 6
4! = (4)(3)(2)(1) = 24
5! = (5)(4)(3)(2)(1) = 120

6! = (6)(5)(4)(3)(2)(1) = 720
Note: 0!=1
Example 2:

Lets say you have four friends, but only need to text three of them when order matters.
Find the number of ways to text your friends.
Solution:

P(4,3) =

4!
24
=
= 24
(4 - 3)! 1!

There are 24 ways to test three out of your four friends if order matters.
Combination Formula:
The permutation of n things taken r at a time is:

n = the total number of items you have from which to choose


r = the number you are actually going to use.
Example 3:

The art club has 4 members. They want to choose a group of three to compete in a
regional competition. How many ways can three members be chosen?
Solution:

There are 4 ways to chose 3 people for the competition when order is not important

The pigeonhole principle (PHP):

The general rule states when there are k pigeonholes and there are k+1 pigeons, then
they will be 1 pigeonhole with at least 2 pigeons. A more advanced version of the
principle will be the following: If mn + 1 pigeons are placed in n pigeonholes, then there
will be at least one pigeonhole with m + 1 or more pigeons in it.
For Example, 13 people are involved in a survey to determine the month of
their birthday. As we all know, there are 12 months in a year, thus, even if the first 12
people have their birthday from the month of January to the month of December, the
13th person has to have his birthday in any of the month of January to December as well.
Thus, by PHP we are right to say that there are at least 2 people who have their birthday
falling in the same month.
In fact, we can view the problem as there are 12 pigeonholes (months of the
year) with 13 pigeons (the 13 persons). Of course, by the Pigeonhole Principle, there
will be at least one pigeonhole with 2 or more pigeons.
PRINCIPLE OF INCLUSION-EXCLUSION:

The Principle of Inclusion and Exclusion allows us to find the cardinality of a


union of sets by knowing the cardinalities of the individual sets and all possible
intersections of them.
The basic version of the Principle of Inclusion and Exclusion is that for two finite sets A
and B, is
|AB|=|A|+|B||AB|.
The result generalizes to three finite sets (in fact it generalizes to any finite number of
finite sets):
|ABC|=|A|+|B|+|C||AB||AC||BC|+|ABC|

Example :
In a room of 50 people whose dresses have either red or white color, 30 are wearing red dress,
16 are wearing a combination of red and white. How many are wearing dresses that have only
white color?

Solution

Number of people wearing a red dress = 30


i.e., n(R) = 30
Number of people wearing a combination of red and white = 16
i.e., n (R

W) = 16

The total number of people in the room = number of people who are wearing dresses
that have either red or white colour = n (R

W) = 50.

We know,
n (R

W) = n(R) + n(W) - n(R

W)

50 = 30 + n(W) - 16
50 - 14 = n(W) - 16
n(W) = 36
i.e., the number of people who are wearing a white dress = 36.
Therefore, number of people who are wearing white dress only = n(W) - n(R

W) =

36 - 16 = 20
Example :

How many members of {1, 2, 3, .., 105} have nontrivial factors in common
with 105?
Solution
105 = 3 . 5. 7, so a number shares factors with 105 if and only if it is divisible by 3, 5,
or 7.
Let A, B, and C be the members of {1, 2, 3, .., 105} divisible by 3, 5, and 7
respectively.
Clearly |A| = 35, |B| = 21, and |C| = 15. Furthermore, A B consists of those numbers
divisible by both and 5, i.e., divisible by 15. Likewise, A C and B C contain
multiples of 21 and 35
respectively, so |A B| = 7, |A C| = 5, and |B C|= 3. Finally, A B C consists
only of the number 105, so it has 1 member total. Thus,
|A U B U C| = 35 + 21 + 15 - 7 - 5 - 3 + 1 = 57

Example:

At Sunnydale High School there are 28 students in algebra class,30 students in biology
class, and 8 students in both classes. How many students are in either algebra or biology
class?
Solution:
Let A denote the set of students in algebra class and B denote the set of students in
biology class. To find the number of students in either class, we first add up the students
in each class:

|A| + |B|
However, this counts the students in both classes twice. Thus we have to subtract them
once:|A B|
This shows
|AUB|=|A| + |B|-|A B|
|AUB|=28 + 30 - 8 = 50
so there are 50 students in at least one of the two classes.
Example:

At Sunnydale High School there are 55 students in either algebra, biology, or chemistry
class 28 students in algebra class, 30 students in biology class, 24 students in chemistry
class, 8 students in both algebra and biology, 16 students in both biology and chemistry,
5 students in both algebra and chemistry. How many students are in all three classes?
Solution:

Let A, B, C denote the set of students in algebra, biology, and chemistry class,
Respectively. Then A U BU C is the set of students in one of the three classes, AB is
the set of students in both algebra and biology, and so forth. To count the number of
Students in all three classes, i.e. count | A U BU C |, we can first add all the number of
students in all three classes:

|A| + |B|+|C|
However, now we've counted the students in two classes too many times. So we subtract
out the students who are in each pair of classes:
-|A B|-|A C|-|B C|
For students who are in two classes, we've counted them twice, then subtracted them
once, so they're counted once. But for students in all three classes, we counted them 3
times, then subtracted them 3 times. Thus we need to add them again:|ABC|
Thus
| A U BU C |=|A| + |B|+|C| -|A B|-|A C|-|B C|+|ABC|
55 = 28 + 30 + 24 - 8 - 16 - 5 + |ABC|
Thus |ABC| = 2, i.e. there are 2 students in all three classes.
RECURRENCE RELATION:

We are familiar with some problem solving techniques for counting,


such as principles for addition, multiplication, permutations, combinations etc.
But there are some problems which cannot be solved or very tedious to solve,
using these techniques. In some such problems, the problems

can be

represented in the form of some relation and can be solved accordingly.


We shall discuss some such examples before proceeding further.
The expression of higher terms in terms of combination of lower terms is
known as recurrence relation
Example: The number of bacteria, double every hour, then what will be the
population of the bacteria after 10 hours? Here we can represent number of
bacteria at the nth hour be an. Then, we can say that an = 2an1.
Example: Consider the Fibonacci sequence
1, 1, 2, 3, 5, 8, 13,
The recurrence relation is given by:
an an1 an2 , a0 a1 1

Example : Towers of Hanoi is a popular puzzle. There are three pegs mounted
on a board, together with disks of different sizes. Initially, these discs are placed
on the first peg in order of different sizes, with the largest disc at the bottom and
the smallest at the top. The task is to move the discs from the first peg to the
third peg using the middle peg as auxiliary. The rules of the puzzle are:
Only one disc can be moved at a time.
No disc can be placed on the top of a smaller disc.
This is a popular puzzle and we shall discuss its solution, using the one of the
techniques discussed in this chapter.
With these illustrations, we define recurrence relation now.
Definition:

recurrence

relation

for

the

sequence

{an}

is

an

equation, that expresses an in terms of one or more of the previous terms of


the sequence, namely, a0, a1, ..., an1, for all integers n with n n0,
where n0 is a nonnegative integer.
Example : an = 1.06an1, with a0 = 0.5.
Example : an = 2an1 + 5, with a0 =1.
The term a0, given in the above two examples, specify initial condition to
solve the recurrence relation completely.
FORMULATION OF RECURRENCE RELATION:

Before we proceed with discussing various methods of solving recurrence


relation, we shall formulate some recurrence relation. The first example of
formulation that we discuss is the problem of Tower of Hanoi as above.
Example: With reference to above Example, let Hn denote the number of

moves required to solve the puzzle with n discs. Let us define Hn


recursively.
Solution: Clearly, H1 = 1.
Consider top (n1) discs. We can move these discs to the middle peg using
Hn1 moves. The nth disc on the first peg can then moved to the third peg. Finally,
(n1) discs from the middle peg can be moved to the third peg with first peg
as auxiliary in Hn1 moves. Thus, total number of moves needed to move n
discs are: Hn = 2Hn1 + 1. Hence the recurrence relation for the Tower of Hanoi is:

Hn = 1

if n = 1.

Hn = 2Hn1 + 1

otherwise.

Example: Find recurrence relation and initial condition for the number of bit
strings of length n that do not have two consecutive 0s.
Solution: Let an denote the number of bit strings of length n that do not

contain two consecutive 0s. Number of bit strings of length one


that follow the necessary rule are: string 0 and string 1. Thus, a1
= 2. The number of bit strings of length 2 is: string 01, 10 and 11.
Thus, a2 = 3. Now we shall consider the case n 3. The bit strings
of length n that do not have two consecutive 0s are precisely those
strings length n1 with no consecutive 0s along with a 1 added 1 at
the end of it (which is an1 in number) and bit strings of length n2
with no consecutive 0s with a 10 added at the end of it (which is
an2 in number). Thus, the recurrence relation is:
an = an1 + an2

for n 3 with a1 = 2 and a2 = 3.

METHODS OF SOLVING RECURRENCE RELATION :

Now, in this section we shall discuss a few methods of solving recurrence relation
and hence solve the relations that we have formulated in the previous section.
Backtracking Method:
This is the most intuitive way of solving a recurrence relation. In this method,
we substitute for every term in the sequence in the form of previous term (i.e. an
in the form of an1, an1 in the form of an2 and so on) till we reach the initial
condition and then substitute for the initial condition.

To

understand

this

better, we shall solve the recurrence relations that we have come across earlier.
Example: Solve the recurrence relation an = 1.06an1, with a0 = 0.5.
Solution: Given recurrence relation is an = 1.06an1, with a0 = 0.5. From this
equation, we have an = 1.06an1 = 1.061.06 an2 = 1.061.061.06
an3 Proceeding this way, we have an = (1.06)na0. But, we know that a0 =
0.5.Thus, explicit solution to the given recurrence relation is an =
n

0.5(1.06) for n 0.

Method for solving linear homogeneous recurrence relations with constant


coefficients:

In the previous subsection, we have seen a backtracking


method for solving

recurrence

relation.

However,

not all

the

equations can be solved easily using this method. In this subsection, we


shall discuss the method of solving a type of recurrence relation called
linear homogeneous recurrence relation. Before that we shall define this
class of recurrence relation.
Definition : A linear homogeneous recurrence relation of degree k

with constant

coefficients is a recurrence relation of the form:a n c1a n 1 c 2 a n 2 c k a n k ,


where c1, c2, ..., ck are constant real numbers with ck 0.

Example :

Fibonacci sequence is also an example of a linear homogeneous


recurrence relation of degree 2.

Example: The recurrence relation a n an-1 not linear (due to ansquare term), whereas the relation Hn = 2Hn1 + 1 is not homogeneous
(due to constant 1).

The basic approach for solving a linear homogeneous recurrence


n
relation to look for the solution of the form an = r , where r is
constant. Note that, rn is a solution to the linear homogeneous
recurrence relation of
degree k, if and only if;
r n c1 r n 1 c 2 r n 2 c k r nk . When both the sides of the
equation are
divided by r

nk

and right side is subtracted from the left side, we

obtain an equation, known as characteristic equation of the recurrence


relation as
follows:
r k c1 r k 1 c2 r k 2 ck 1 r ck 0 .
The solutions of the equation are called as characteristic roots of
the recurrence relation.

In this subsection, we shall focus on solving linear homogeneous


recurrence relation of degree 2 that is: an = c1an1 + c2an2.
The characteristic equation of this relation is r2 c1r c2 = 0. This is a
quadratic equation and has two roots. Two cases arise.
(i) Roots are distinct, say s1 and s2. Then, it can be shown that
a n us1n vs2n is a solution to the recurrence relation, with

a1 us1 vs 2 and a 2 us12 vs22 .

(ii)

Roots are equal, say s. Then it can be shown that an solution to the

recurrence relation is an= (u vn)s

We shall use above results to solve some problems


Example : Solve the recurrence relation bn + 3bn1 + 2bn2 = 0, with b1 = 2 and
b2 = 4.
Solution: The characteristic equation to the given recurrence relation is x2

+ 3x + 2 = 0. Roots of this equation are s1 = 2 and s2 = 1.


Hence the solution to the relation is:
bn = u(1)n + v(2)n. b1 = 2 = u 2v and b2 = 4 = u + 4v.
Solving these two equations simultaneously, we get, u = 0 and v = 1.
n
Thus, explicit solution to the given recurrence relation is bn = (2)
Method for solving linear non-homogeneous recurrence relations with
constant coefficients:

The method is similar to the solution differential equation by method of


undermined co-efficient.

GENERATING FUNCTION:

Let a0 , a1 ,...an be a sequence, and then the corresponding generating function is given
by:
A(x) = a0 x 0 a1 x1 ... an x n
For Example, if 1, 1, 1,. be a sequence then the corresponding generating function is
given by:
A(x) = 1 x x 2 .... 1 /(1 x)

From a given sequence we can find the corresponding generating function and vice
versa.

Unit II
INTRODUCTION TO RELATIONS AND
GRAPH THEORY
OBJECTIVES:
After going through this unit, you will be able to know:

Definition of Relation.

Representation of Relations

Types of Relations

Equivalence of relations

Relations and Partition

Definition and examples of partial order relation

Representation of posets using Hasse diagram

Closure of relations.

Introduction to graphs

Graph terminology

Graph isomorphism

Connectivity

Euler and Hamilton paths

Planar graphs

Graph colouring

Introduction to trees

INTRODUCTION :
Relationships between elements of sets occur in many
contexts. We deal with many relationships such as students name
and roll no., teacher and her specialization, a person and a relative
(brother sister, mother child etc.). In this section, we will discuss
mathematical approach to the relation. These have wide applications in
Computer science (e.g. relational algebra)

RELATIONS:
Relationship between elements of sets is represented using a mathematical structure
called relation. The most intuitive way to describe the relationship is to represent
in the form of ordered pair. In this section, we study the basic terminology
and diagrammatic representation of relation.
Definition :

Let A and B be two sets. A binary relation from A to B is a subset of A B.


Note : If A, B and C are three sets, then a subset of ABC is known as ternary
relation. Continuing this way a subset of A1A2...An is known as n ary
relation.
Note: Unless or otherwise specified in this chapter a relation is a binary relation.

Let A and B be two sets. Suppose R is a relation from A to B (i.e. R is a


subset of A B). Then, R is a set of ordered pairs where each first element
comes from A and each second element from B. Thus, we denote it with an
ordered pair (a, b), where a A and b B. We also denote the relationship with
a R b, which is read as a related to b. The domain of R is the set of all first
elements in the ordered pair and the range of R is the set of all second elements
in the ordered pair.
Example 1: Let A = { 1, 2, 3, 4 } and B = { x, y, z }. Let R = {(1, x), (2, x), (3, y), (3, z)}.
Then R is a relation from A to B.
Example 2: Suppose we say that two countries are adjacent if they have some part
of their boundaries common. Then, is adjacent to, is a relation R on the
countries on the earth. Thus, we have, (India, Nepal) R, but (Japan, Sri
Lanka) R.
Example 3: A familiar relation on the set Z of integers is m divides n. Thus,

we have, (6, 30) R, but (5, 18) R.


Example 4: Let A be any set. Then A A and are subsets of A A and hence

they are relations from A to A. These are known as universal


relation and empty relation, respectively.
Note : As relation is a set, it follows all the algebraic operations on relations that we
have discussed earlier.

Definition : Let R be any relation from a set A to set B. The inverse of R,

denoted by R1, is the relation from B to A which consists of those


ordered pairs, when reversed, belong to R. That is:
R1 = {(b, a) : (a, b) R}
Example 5:
Inverse relation of the relation in example 1 is, R1 = {(x,), (x, 2), (y, 3), (z, 3)}.

REPRESENTATION OF RELATIONS:
Matrices and graphs are two very good tools to represent various
algebraic structures. Matrices can be easily used to represent relation in
any

programming

language

in

computer.

Here we

representation of relation on finite sets using these tools.


Consider the relation in Example1.

x y

1 1

0 0

2 1

0 0

3 0 1 1
4 0 0 0

Fig. 1

discuss

the

Thus, if a R b, then we enter 1 in the cell (a, b) and 0 otherwise. Same relation can
be represented pictorially as well, as follows:

1
2
3
4

y
z
Fig 2

Thus, two ovals represent sets A and B respectively and we draw an arrow from
a A to b B, if a R b.
If the relation is from a finite set to itself, there is another way of pictorial representation,
known as diagraph.
For example, let A = {1, 2, 3, 4} and R be a relation from A to itself, defined
as follows:

R = {(1, 2), (2, 2), (2, 4), (3, 2), (3, 4), (4, 1), (4, 3)} Then, the diagraph of R is
drawn as follows:

4
Fig 3

The directed graphs are very important data structures that have
applications in Computer Science (in the area of networking).
Definition : Let A, B and C be three sets. Let R be a relation from A to B and S
be a relation from B to C. Then, composite relation RS, is a
relation from A to C, defined by, a(RS)c, if there is some b B, such
that a R b and b S c.

Example 6: Let A = {1, 2, 3, 4}, B = {a, b, c, d},C = {x, y, z } and let R = {(1, a), (2, d),
(3, a), (3, b), (3, d)} and S = {(b, x), (b, z), (c, y), (d, z)}.

Pictorial representation of the relation in Example 6 can be shown as


below (Fig 4).

1
2
3
4

a
b
c
d

x
y
z

Fig.4
Thus, from the definition of composite relation and also from Fig 4, RS
will be given as below.
RS = {(2, z), (3, x), (3, z)}.
There is another way of finding composite relation, which is using
matrices.
Example7: Consider
as follows.
1 0

0 0
MR
1 1

relations R and S in Example 6. Their matrix representations are


0 0

0 1
0 1

0 0 0

1 0 1
MS
0 1 0

0 0 0 0

0 0 1


Consider the product of matrices MR and MS as follows: Observe that the non-zero entries
in the product tell us which elements are related in RS. Hence, MRMS and MRS have
same non-zero entries.

TYPES OF RELATIONS:
In this section, we discuss a number of important types of relations defined from a set
A to itself.
Definition : Let R be a relation from a set A to itself. R is said to be reflexive, if for
every a A, a R a (a is related to itself).
Example 8: Let A = {a, b, c, d} and R be defined as follows: R = {(a, a), (a, c), (b, a), (b, b),
(c, c), (d, c), (d, d)}. R is a reflexive relation.
Example 9: Let A be a set of positive integers and R be a relation on it defined as,
a R b if a divides b. Then, R is a reflexive relation, as a
divides to itself for every positive integer a.
Note : If we draw a diagraph of a reflexive relation, then all the vertices will have a
loop.

Also

if we represent

reflexive relation using a matrix, then all its

diagonal entries will be 1.


Definition : Let R be a relation from a set A to itself. R is said to be irreflexive,
if for every a A, a R a
Example 10: Let A be a set of positive integers and R be a relation on it defined as,
a R b if a is less than b. Then, R is an irreflexive relation, as a is not less than
itself for any positive integer a.
Example 11: Let A = {a, b, c, d} and R be defined as follows: R = {(a, a), (a, c), (b, a), (b,
d), (c, c), (d, c), (d, d)}.Here R is neither reflexive nor irreflexive relation as b
is not related to itself and a, c, d are related to themselves.
Note : If we draw a diagraph of an irreflexive relation, then no vertex will have a loop.
Also if we represent irreflexive relation using a matrix, then all its diagonal
entries will be 0.
Definition : Let R be a relation from a set A to itself. R is said to be symmetric, if for
a, b A, if a R b then b R a.
Definition : Let R be a relation from a set A to itself. R is said to be anti-symmetric, if for
a, b A, if a R b and b R a, then a = b. Thus, R is not anti-symmetric
if there exists a, b A such that a R b and b R a but a b.
Example 13: Let A = {1, 2, 3, 4} and R be defined as:
R = {(1, 2), (2, 3), (2, 1), (3, 2), (3, 3)}, then R is symmetric relation.
Example 14: An equality (or is equal to) is a symmetric relation on the set of
integers.

Example 15: Let A = {a, b, c, d} and R be defined as: R = {(a, b), (b, a), (a, c), (c, d),
(d, b)}. R is not symmetric, as a R c but c R a . R is not anti-symmetric, because
a R b and b R c , but a b.
.Example 16: The relation less than or equal to (), is an anti- symmetric
relation.
Example 17: Relation is less than ( < ), defined on the set of all real numbers, is an
asymmetric relation.
Definition : Let R be a relation defined from a set A to itself. R is said to transitive, if for
a, b, c A, a R b and b R c, then a R c.
Example 18: Let A = {a, b, c, d} and R be defined as follows: R = {(a,b), (a, c), (b, d),
(a, d), (b, c), (d, c)}. Here R is transitive relation on A.
Example 19: Relation a divides b, on the set of integers, is a transitive relation.
Definition : Let R be a relation defined from a set A to itself. If R is reflexive, symmetric
and transitive, then R is called as equivalence relation.
Example 20: Consider the set L of lines in the Euclidean plane. Two lines in the plane
are said to be related, if they are parallel to each other. This relation is an
equivalence relation.
Example 21: Let m be a fixed positive integer. Two integers, a, b are said to be
congruent modulo m, written as: a b (mod m), if m divides a b. The
congruence relation is an equivalence relation.
Example 22 : Let A2,3,4,5 and let R 2,3 , 3,3 , 4,5 , 5,1 . Is R symmetric,
asymmetric or antisymmetric

Solution :
a)

R is not symmetric, since 2, 3 R , but 3, 2 R ,

R is not asymmetric since 3, 3 R


R is antisymmetric.

Example 23 : Determine whether the relation R on a set A is reflexive, irreflexire,


symmetric, asymmetric antisymmetric or transitive.
b)
c)

I)

A = set of all positive integers, a R b iff a b 2 .

Solution :
1)
R is reflexive because a a 02,a A
2)

R is not irreflexive because 1 102 for 1A (A is the set

3)

of all positive integers.)


R is symmetric because a b 2 b a 2 aRbbRa

4)

R is not asymmetric because 5 4 2 and we have 4 5 2

5)

5R44R5
R is not antisymmetric because 1R2 & 2R1 1R21 2 2 &
2R1 2 12 . But 12

6)
II)

R is not transitive because 5 R 4, 4 R 2 but 5 R 2


AZ ,aRb iff a b 2

Solution :
As per above example we can prove that R is not reflexive, R is
irreflexive, symmetric, not asymmetric, not antisymmetric & not transitive
III)
1)

Let A = {1, 2, 3, 4} and R {(1,1), (2,2), (3,3)}


R is not reflexive because 4, 4 R

2)
3)
4)

R is not irreflexive because 1,1R


R is symmetric because whenever a R b then b R a.
R is not asymmetric because R R

5)
R is antisymmetric because 2R2,2R222
6)
R is transitive.
IV)
Let AZ ,aRb iff GCD (a, b) = 1 we can say that a and b are
relatively prime.
1)
2)

R is not reflexive because 3, 3 1 it is 3. 3, 3 R


R is not irreflexive because (1, 1) = 1

3)
4)
5)
6)

R is symmetric because for a, b 1 b, a 1 . aRbbRa


R is not asymmetric because (a, b) = 1 then (b, a) = 1.
aRbbRa
R is not antisymmetric because 2 R 3 and 3 R 2 but 23 .
R is not transitive because 4 R 3, 3 R 2 but 4 R 2 because
(4,2) = G.C.D. (4,2) = 21 .

V)

A = Z a R b iff ab1

1)
2)
3)
4)
5)
6)

R is reflexive because aa1 a| A .


R is not irreflexive because 001 for .
R is not symmetric because for 251 does not imply 521.
R is not asymmetric because for (2,3) R and also (3,2) R.
R is not antisymmetric because 5 R 4 and 4 R 5 but 45 .
R is not transitive because (6,45) R, (5,4) R but (6,47) R.

RELATIONS AND PARTITION:


In this section, we shall know what partitions are and its relationship
with equivalence relations.
Definition : A partition or a quotient set of a non-empty set A is a collection P
of non-empty sets of A, such that
(i) Each element of A belongs to one of the sets in P.
(ii) If A1 and A2 are distinct elements of P, then A1A2 = .
The sets in P are called the blocks or cells of the partition.
Example : Let A = {1, 2, 3, 4, 5}. The following sets form a partition of A, as A =
A1 A2 A3 and A1 A1 andA2
A1 = {1, 2}; A2 = {3, 5}; A3 = {4}.
Example 24: Let A = {1, 2, 3, 4, 5, 6}. The following sets do not form a partition of A, as
A = A1 A2 A3 but A2 A1 = {1, 2}; A2 = {3, 5}; A3 = {4, 5, 6}.
The following result shows that if P is a partition of a set A, then P can be
used to construct an equivalence relation on A.
Theorem: Let P be a partition of a set A. Define a relation R on A as a R b if and only if
a, b belong to the same block of P then R is an equivalence relation on A.

Example 25: Consider the partition defined in Example 23. Then the equivalence
relation as defined from the partition is:
R={(1, 1),(1, 2),(2, 1),(2, 2),(3, 3),(3, 5), (5, 3), (5, 5), (4, 4)}.
Now, we shall define equivalence classes of R on a set A.
Theorem: Let R be an equivalence relation on a set A and let a, b A, then a R b if and
only if R(a) = R(b), where R(a) is defined as: R(a) = {x A: a R x}. R(a) is
called as relative set of a.
Example26: If we consider an example in 25, we observe that, R(1) = R(2), R(3) = R(5).
Because R (1) = {1,2}, R (2) = {1,2}, R (3) = {3,5}, R(5) = {3,5}.
Earlier, we have seen that, a partition defines an equivalence relation. Now,
we shall see that, an equivalence relation defines a partition.
Theorem: Let R be an equivalence relation on A and let P be the collection of all distinct
relative sets R(a) for a A. Then P is a partition of A and R is equivalence
relation of this partition.
Note: If R is an equivalence relation on A, then sets R(a) are called as equivalence
classes of R.
Example 27: Let A = {1, 2, 3, 4} and R = {(1, 1), (1, 2), (2, 1), (2, 2), (3,4), (4, 3), (3, 3),
(4, 4)}. We observe that R(1) = R(2) and R(3) = R(4) and hence P = { {1, 2}, {3,
4} }.
Example 28: Let A = Z (set of integers) and define R as
R = {(a, b) A A: a b (mod 5)}. Then, we have,
R(1) = {......,14, 9, 4, 1, 6, 11, ..... }
R(2) = {......,13, 8, 3, 2, 7, 12, ..... }
R(3) = {......,12, 7, 2, 3, 8, 13, ..... }
R(4) = {......,11, 6, 1, 4, 9, 14, ..... }
R(5) = {......,10, 5, 0, 5, 10, 15, ..... }.

R(1), R(2), R(3), R(4) and R(5) form partition on Z with respect to given
equivalence relation.

PARTIAL ORDER RELATION


We often use relation to describe certain ordering on the sets. For example,
lexicographical ordering is used for dictionary as well as phone directory. We
schedule c e r t a i n j o b s as per certain ordering, such as priority. Ordering of numbers
may be in the increasing order.

In the previous chapter, we have discussed various properties (reflexive etc) of


relation. In this chapter we use these to define ordering of the sets.

Definition 1: A relation R on the set A is said to be partial order relation, if it is


reflexive, anti-symmetric and transitive.
Before we proceed further, we shall have a look at a few examples of partial order
relations.
Example 1: Let A = {a, b, c, d, e}. Relation R, represented using following
matrix is a partial order relation.

0
0

0
0

1 1 1 1

1 1 1 1
0 1 1 1

0 0 1 1
0 0 0 1

Observe the reflexive, anti-symmetric and transitive properties of


the relation from the matrix.
Example 2: Let A be a set of natural numbers and relation R be less than or equal to
relation (). Then R is a partial order relation on A. For any m, n, k
N, n n (reflexive); if
m n and m n, then m = n (antisymmetric); lastly, if m n and n k, then m k (transitive).
Definition : If R is a partial order relation on a set A, then A is called as partial order set
and it is denoted with (A, R). Typically this set is termed as poset and
the pair is denoted with (A, ).
DIAGRAMMATIC REPRESENTATION OF PARTIAL ORDER RELATIONS
AND POSETS:
In the previous chapter, we have seen the diagraph of a relation. In this section, we use the

diagraphs of the partial order relations, to represent the relations in a very suitable way
where there no arrowhead and transitivity shown indirectly known as Hasse diagram.
We understand the Hasse diagram, using following example.
Example 1: Let A = {a, b, c, d, e} and the following diagram represents the diagraph of
the partial order relation on A.

a
Fig.1

Now, we shall draw Hasse diagram from the above diagrams using following rules.
(i) Drop the reflexive loops
c

a
Fig. 2

(ii) Drop transitive lines

a
Fig. 3

(iii)Drop arrows
c

a
Fig.4

Note : In many cases, when the graphical representation is so oriented that all the arrow
heads point in one direction (upward, downward, left to right or right to left). A
graphical representation in which all the arrowheads point upwards, is known as
Hasse diagram.

Example 4: `Let A = {1, 2, 3, 4, 6, 9} and relation R defined on A be a divides b. Hasse


diagram for this relation is as follows:

Note : The reader is advised to verify that this relation is indeed a partial order relation.
Further, arrive at the following Hasse diagram from the diagraph of a relation as
per the rules defined earlier.

6
4

Fig.5
Example 5 : Determine the Hasse diagram of the relation on A = {1,2,3,4,5}
whose MR is given below :
10111
01111

M R 0011

0
00001

Solution :
Reflexivity is represented

by 1 at diagonal

place.

So after removing

reflexivity R is R = {(1,3), (1,4), (1,5), (2,3), (2,4), (2,5), (3,4), (3,5)}


Remove transitivity as


1, 3 3, 4 R
remove 1, 4 R
2, 3 3, 5 R remove 2, 5R
R1, 3 , 2, 3 , 3, 4 , 3, 5

and so on.

The Hasse Diagram is


5

1
Example 6 :

Determine matrix of partial order whose Hasse diagram is given as follow -

1
Solution :
Here A = [1, 2, 3, 4, 5)
Write all ordered pairs (a, a) a A i.e. relation is reflexive.

Then write all ordered pairs in upward direction. As (1, 2) R & (2,4) R1, 4 R since
R is transitive.

R1,1,2,2,3,3,4,4,5,5,1,2,2,4,2,4,1,4 ,1,3,3,5,1,5

The matrix MR can be written as -

MR

1
0

1
1
0
0
0

1
0
1
0
0

1
1
0
1
0

1
0

0
1

Now, we shall have a look at certain terms with reference to posets.


Definition : Let (A, ) be a partially ordered set. Elements a, b A, are said to be comparable, if a b
or b a.
E.g. In example 4, 2 and 4 are comparable, whereas 4 and 9 are not
comparable.
Definition : Let (A, ) be a partially ordered set. A subset of A is said to be a chain if every two
elements in the subset are related.
Example 7: In the poset of example 4, subsets {1, 2, 4}; {1, 3, 6};{1, 2, 6} and {1, 3, 9}
are chains.
Definition : A subset of a poset A is said to be anti-chain, if no two elements of it are related.
Example 8: In the poset of example 4, subsets {2, 9}; {3, 4}; {4, 6, 9}are anti-chains.
Definition : A partially ordered set A is said to be totally ordered if it is chain.
Example 9: Let A = {2, 3, 5, 7, 11, 13, 17, 19} and the relation defined on A be .
Then poset (A, ) is a chain.
CLOSURE PROPERTIES

Consider a given set A and let R be a relation on A. Let P be a property of such relations,
such as being reflexive or symmetric or transitive. A relation with property P will be called
a P-relation. The P-closure of an arbitrary relation R on A, written P (R), is a P-relation such
that

R P (R) S
for every P-relation S containing R. We will write
reexive (R), symmetric(R), and transitive(R)
for the reexive, symmetric, and transitive closures of R.
Generally speaking, P (R) need not exist. However, there is a general situation where P (R)
will always exist. Suppose P is a property such that there is at least one P-relation containing
R and that the intersection of any P-relations is again a P-relation. Then one can prove that
P (R) = (S | S is a P -relation and R S)
Thus one can obtain P (R) from the top-down, that is, as the intersection of relations. However,
one usually wants to nd P (R) from the bottom-up, that is, by adjoining elements to R to obtain
P (R). This we do below.
Reexive and Symmetric Closures
The next theorem tells us how to obtain easily the reexive and symmetric closures of a
relation. Here
A = {(a, a) | a A} is the diagonal or equality relation on A.
Theorem: Let R be a relation on a set A. Then:
(i) R A is the reexive closure of R.
(ii) R R1 is the symmetric closure of R.
In other words, reexive(R) is obtained by simply adding to R those elements (a, a) in the
diagonal which do not already belong to R, and symmetric(R) is obtained by adding to R all pairs
(b, a) whenever (a, b) belongs to R.

EXAMPLE 10
set A = {1, 2, 3, 4}.

Consider the relation R = {(1, 1), (1, 3), (2, 4), (3, 1), (3, 3), (4, 3)} on the

Then
reexive(R) = R {(2, 2), (4, 4)} and symmetric(R) = R {(4, 2), (3, 4)}
Transitive Closure
2
n
Let R be a relation on a set A. Recall that R = RR and R = R n1 R. We dene
The following theorem applies:
Theorem : R is the transitive closure of R.

Suppose A is a nite set with n elements. We show


2

R = R R . . . R

This gives us the following theorem:


Theorem : Let R be a relation on a set A with n elements. Then
2

transitive (R) = R R . . . R

EXAMPLE 11

Consider the relation R = {(1, 2), (2, 3), (3, 3)} on A = {1, 2, 3}.

Then:
2

R = R R = {(1, 3), (2, 3), (3, 3)} and R = R R = {(1, 3), (2, 3), (3, 3)}
Accordingly,
transitive (R) = {(1, 2), (2, 3), (3, 3), (1, 3)}

MAXIMAL, MINIMAL

ELEMENTS AND LATTICES:

In this section, we discuss certain element types in the poset and


hence a special kind of poset, Lattice.
To understand these types, we shall refer to the following figures,
i.e. Fig.6 and Fig.7.
j

h
g
f
i

b
c
f

g
a

d
a
Fig. 6

e
Fig. 7

Definition : Let (A, ) be a poset. An element a A is called a maximal


element, if for no b A, a b, a b. E.g. In Fig. 4, j and k are maximal
elements.
Definition : Let (A, ) be a poset. An element a A is called a minimal element, if for no
b A, a b, b a. E.g. In Fig. 4.6, a, b and e are minimal elements.
Definition : Let a, b be two elements in the poset (A, ). An element c A, is said to be
an upper bound of a, b if a c and b c. E.g. In Fig 7, f1 h are upper bounds
of b and d.
Definition : Let a, b be two elements in the poset (A, ). An element c A, is said to be a
least upper bound of a, b if a c and b c and if d is an upper bound of a, b,
then c d. E.g. In Fig 2, f is a least upper bound of b and d.
Definition : Let a, b be two elements in the poset (A, ). An element c A, is said to be
a lower bound of a, b if c a and c b. E.g. In Fig 6, f, g are lower bounds
of h and i.
Definition : Let a, b be two elements in the poset (A, ). An element c A, is said to be
a greatest lower bound of a, b if c a and c b and if d is a lower bound of a,
b, then d c. E.g. In Fig 4, c is a greatest lower bound of e and g.
Definition : A poset (A, ) is said to be a lattice, if every two elements in A have a
unique least upper bound and a unique greatest lower bound.
E.g. Fig. 6 is not a lattice, because j and k are two least upper bounds of h and i, whereas
Fig. 7 is a lattice.

Graph Theory
Graphs with Basic Terminology
The fundamental concept of graph theory is the graph, which (despite the name) is best
thought of as a mathematical object rather than a diagram, even though graphs have a
very natural graphical representation. A graph usually denoted G(V,E) or G = (V,E)
consists of set of vertices V together with a set of edges E. Vertices are also known as
nodes, points and (in social networks) as actors, agents or players. Edges are also
known as lines and (in social networks) as ties or links. An edge e = (u,v) is defined by
the unordered pair of vertices that serve as its end points. Two vertices u and v are
adjacent if there exists an edge (u,v) that connects them. An edge e = (u,u) that links a
vertex to itself is known as a self-loop or reflexive tie. The number of vertices in a graph
is usually denoted n while the number of edges is usually denoted m.

As an example, the graph depicted in Figure 1 has vertex set V={a,b,c,d,e.f} and edge
set E = {(a,b),(b,c),(c,d),(c,e),(d,e),(e,f)}.

Figure 1.

When looking at visualizations of graphs such as Figure 1, it is important to realize that


the only information contained in the diagram is adjacency; the position of nodes in the
plane (and therefore the length of lines) is arbitrary unless otherwise specified. Hence it
is usually dangerous to draw conclusions based on the spatial position of the nodes. For
example, it is tempting to conclude that nodes in the middle of a diagram are more
important than nodes on the peripheries, but this will often if not usually be a
mistake.

When used to represent social networks, we typically use each line to represent
instances of the same social relation, so that if (a,b) indicates a friendship between the
person located at node a and the person located at node b, then (d,e) indicates a
friendship between d and e. Thus, each distinct social relation that is empirically
measured on the same group of people is represented by separate graphs, which are
likely to have different structures (after all, who talks to whom is not the same as who
dislikes whom).
Every graph has associated with it an adjacency matrix, which is a binary nn matrix A
in which aij = 1 and aji = 1 if vertex vi is adjacent to vertex vj, and aij = 0 and aji = 0
otherwise. The natural graphical representation of an adjacency matrix is a table, such as
shown in Figure 2.
a b c d e f
a 0 1 0 0 0 0
B 1 0 1 0 0 0
c 0 1 0 1 1 0
D 0 0 1 0 1 0
e 0 0 1 1 0 1
f 0 0 0 0 1 0

Figure 2. Adjacency matrix for graph in Figure 1.

Examining either Figure 1 or Figure 2, we can see that not every vertex is adjacent to
every other. A graph in which all vertices are adjacent to all others is said to be
complete. The extent to which a graph is complete is indicated by its density, which is
defined as the number of edges divided by the number possible. If self-loops are
excluded, then the number possible is n(n-1)/2. If self-loops are allowed, then the
number possible is n(n+1)/2. Hence the density of the graph in Figure 1 is 6/15 = 0.40.
A clique is a maximal complete subgraph. A subgraph of a graph G is a graph whose
points and lines are contained in G. A complete subgraph of G is a section of G that is
complete (i.e., has density = 1). A maximal complete subgraph is a subgraph of G that is
complete and is maximal in the sense that no other node of G could be added to the
subgraph without losing the completeness property. In Figure 1, the nodes {c,d,e}

together with the lines connecting them form a clique. Cliques have been seen as a way
to represent what social scientists have called primary groups.
hile not every vertex in the graph in Figure 1 is adjacent, one can construct a sequence
of adjacent vertices from any vertex to any other. Graphs with this property are called
connected. Similarly, any pair of vertices in which one vertex can reach the other via a
sequence of adjacent vertices is called reachable. If we determine reachability for every
pair of vertices, we can construct a reachability matrix R such as depicted in Figure 3.
The matrix R can be thought of as the result of applying transitive closure to the
adjacency matrix A.

c
g

Figure 3.

A component of a graph is defined as a maximal subgraph in which a path exists from


every node to every other (i.e., they are mutually reachable). The size of a component is
defined as the number of nodes it contains. A connected graph has only one component.
A sequence of adjacent vertices v0,v1,,vn is known as a walk. In Figure 3, the sequence
a,b,c,b,a,c is a walk. A walk can also be seen as a sequence of incident edges, where two
edges are said to be incident if they share exactly one vertex. A walk in which no vertex
occurs more than once is known as a path. In Figure 3, the sequence a,b,c,d,e,f is a path.
A walk in which no edge occurs more than once is known as a trail. In Figure 3, the
sequence a,b,c,e,d,c,g is a trail but not a path. Every path is a trail, and every trail is a
walk. A walk is closed if vo = vn. A cycle can be defined as a closed path in which n >=
3. The sequence c,e,d in Figure 3 is a cycle. A tree is a connected graph that contains no
cycles. In a tree, every pair of points is connected by a unique path. That is, there is only
one way to get from A to B.
The length of a walk (and therefore a path or trail) is defined as the number of edges it
contains. For example, in Figure 3, the path a,b,c,d,e has length 4. A walk between two
vertices whose length is as short as any other walk connecting the same pair of vertices

is called a geodesic. Of course, all geodesics are paths. Geodesics are not necessarily
unique. From vertex a to vertex f in Figure 1, there are two geodesics: a,b,c,d,e,f and
a,b,c,g,e,f.
The graph-theoretic distance (usually shortened to just distance) between two vertices is
defined as the length of a geodesic that connects them. If we compute the distance between
every pair of vertices, we can construct a distance matrix D such as depicted in Figure 4. The
maximum distance in a graph defines the graphs diameter. As shown in Figure 4, the diameter
of the graph in Figure 1 is 4. If the graph is not connected, then there exist pairs of vertices
that are not mutually reachable so that the distance between them is not defined and the
diameter of such a graph is also not defined.
a B c d e f G
a 0 1 2 3 3 4 3
b 1 0 1 2 2 3 2
c

2 1 0 1 1 2 1

d 3 2 1 0 1 2 2
e 3 2 1 1 0 1 1
f

4 3 2 2 1 0 2

g 3 2 1 2 1 2 0

Figure 4. Distance matrix for graph in Figure 3.

The powers of a graphs adjacency matrix, Ap, give the number of walks of length p
between all pairs of nodes. For example, A2, obtained by multiplying the matrix by
itself, has entries aij2 that give the number of walks of length 2 that join node vi to node
vj. Hence, the geodesic distance matrix D has entries dij = p, where p is the smallest p
such that aijp > 0. (However, there exist much faster algorithms for computing the
distance matrix.)
The eccentricity e(v) of a point v in a connected graph G(V,E) is max d(u,v), for all u V.
In other words, a points eccentricity is equal to the distance from itself to the point
farthest away. The eccentricity of node b in Figure 3 is 3. The minimum eccentricity of
all points in a graph is called the radius r(G) of the graph, while the maximum
eccentricity is the diameter of the graph. In Figure 3, the radius is 2 and the diameter is
4. A vertex that is least distant from all other vertices (in the sense that its eccentricity

equals the radius of the graph) is a member of the center of the graph and is called a
central point. Every tree has a center consisting of either one point or two adjacent
points.

Directed Graphs
As noted at the outset, the edges contained in graphs are unordered pairs of nodes (i.e.,
(u,v) is the same thing as (v,u)). As such, graphs are useful for encoding directionless
relationships such as the social relation sibling of or the physical relation is near.
However, many relations that we would like to model are not directionless. For
example, is the boss of is usually anti-symmetric in the sense that if u is the boss of v,
it is unlikely that v is the boss of u. Other relations, such as gives advice to are simply
non-symmetric in the sense that if u gives advice to v, v may or may not give advice to
u.
To model non-symmetric relations we use directed graphs, also known as digraphs. A
digraph D(V,E) consists of a set of nodes V and a set of ordered pairs of nodes E called
arcs or directed lines. The arc (u,v) points from u to v.

Figure 5a

b
c

e
f

Figure 5b

Digraphs are usually represented visually like graphs, except that arrowheads are placed
on lines to indicate direction (see Figure 5). When both arcs (u,v) and (v,u) are present in
a digraph, they may be represented by a double-headed arrow (as in Figure 5a), or two
separate arrows (as shown in Figure 5b).

In a digraph, a walk is a sequence of nodes vo,v1,vn in which each pair of nodes vi,
vi+1 is linked by an arc (vi,vi+1). In other words, it is a traversal of the graph in which
the flow of movement follows the direction of the arcs, like a car moving from place to
place via one-way streets. A path in a digraph is a walk in which all points are distinct.
A semiwalk is a sequence of nodes vo,v1,vn in which each pair of nodes vi, vi+1 is
linked by either the arc (vi,vi+1) or the arc (vi+1,vi). In other words, in a semiwalk, the
traversal need not respect the direction of arcs, like a car that freely goes the wrong way
on one-way streets.

By analogy, we can also define a semipath, semitrail, and

semicycle.
Another way to think of semiwalks is as walks on the underlying graph, where the
underlying graph is the graph G(V,E) that is formed from the digraph D(V,E) such that
(u,v) E if and only if (u,v) E or (v,u) E. Thus, the underlying graph of a digraph
is basically the graph formed by ignoring directionality.
A digraph is strongly connected if there exists a path (not a semipath) from every point
to every other. Note that the path from u to v need not involve the same intermediaries
as the path from v to u. A digraph is unilaterally connected if for every pair of points
there is a path from one to the other (but not necessarily the other way around). A
digraph is weakly connected if every pair of points is mutually reachable via a semipath
(i.e., if the underlying graph is connected).
A strong component of a digraph is a maximal strongly connected subgraph. In other
words, it is a subgraph that is strongly connected and which is as large as possible (there
is no node outside the subgraph that is strongly connected to all the nodes in the
subgraph). A weak component is a maximal weakly connected subgraph.
The number of arcs originating from a node v (i.e., outgoing arcs) is called the outdegree
of v, denoted od(v). The number of arcs pointing to a node v (i.e., incoming arcs) is
called the indegree of v, denoted id(v). In a graph representing friendship feelings

among a set of persons, outdegree can be seen as indicating gregariousness, while


indegree corresponds to popularity. The average outdegree of a digraph is necessarily
equal to the average indegree.
The adjacency matrix A of a digraph is an n n matrix in which aij = 1 if (vi,vj) E and
aij = 0 otherwise. Unlike the adjacency matrix of an undirected graph, the adjacency
matrix of a directed graph is not constrained to be symmetric, so that the top right half
need not equal the bottom left half (i.e., aij <> aji). If a digraph is acyclic, then it is
possible to order the points of D so that the adjacency matrix upper triangular (i.e., all
positive entries are above the main diagonal).
Some notations

Kn : the complete graph on n vertices.


Cn : the n-cycle graph.
Km,n : the complete bipartite graph on m+n vertices and mn edges..
K1,n : the star graph on n+1 vertices.
mKn : m disjoint copies of Kn.
Paths and Circuits

chain : A sequence of vertices

[v0 , v1, v2 ,... vl ] is a chain of length l in G if

vi 1vi E or vi vi 1 E for i=1,2, ...,l.


path : A sequence of vertices [v0 , v1, v2 ,... vl ] is a path from v0 to vl of length l
in G if vi 1vi

for i=1,2, ...,l.

simple path: It does not include the same edge twice.


elementary path(or chain): A path or chain in G is called elementary if no vertex
occurs more than once.
connected graph : A graph G is connected if between any two vertices there exists a
path in G joining them.
strongly connected graph : A graph G is strongly connected if for any two vertices x
and y there exists a path in G from x to y.
elementary cycle(circuit) : A cycle [v0 , v1, v2 ,... vl , v0 ] is a elementary cycle if
vi vj for ij.

chordless cycle : A simple cycle [v0 , v1, v2 ,... vl , v0 ] is chordless if vi vjE for
i and j differing by more than 1 mod l+1.
Theorem : In a (directed or undirected) graph with n vertices, if there is a path from
vertex v1 to vertex v2, then there is a path of no more than n-1 edges from v1 to
vertex v2.
bipartite graph : An undirected graph G=(V,E) is bipartite if its vertices can be
partitioned into two disjoint stable sets V=S1+S2.
complete bipartite graph : A bipartite graph G=(S1,S2,E) is complete if for every xS1
and yS2 we have xyE, i.e., every possible edge that could exist does exist.
Eulerian Paths and Circuits

L. Euler, the father of the graph theorysolved the Knigsbergs bridge problem,
1736
Eulerian path problem : a path that traverses each edge in the graph once and only
once.
Theorem: An undirected graph possess an Eulerian path if and only if it is
connected and has either zero or two vertices of odd degree.
Proof. () Suppose that the graph possess an Eulerian path. It must be connected.
When the eulerian path is traced, we observe that every time the path meets a
vertex, it goes through two edges which are incident with the vertex and have
not been traced before.
Thus, except for the two vertices at the ends of the path, the degree of any
vertex in the graph must be even.
() omitted.
Theorem: An undirected graph possess an Eulerian circuit if and only if it is
connected and has no vertices of odd degree.
Theorem : An directed graph possess an Eulerian circuit if and only if it is
connected and the incoming degree of every vertex is equal to its outgoing degree.

An directed graph possess an eulerian path if and only if it is connected and the
incoming degree of every vertex is equal to its outgoing degree with the possible
exception of two vertices. For these two vertices, the incoming degree of one is one
larger than its outgoing degree, and the incoming degree of the other is one less than
its outgoing degree.

Hamiltonian Paths and Circuits

Hamiltonian path : A path that passes through each of the vertices in a graph exactly
once.
No simple necessary and sufficient condition is known for graph to have a
Hamiltonian path or circuit.
Theorem : Let G be a linear graph of n vertices. If the sum of the degrees for each
pair of vertices in G is n - 1 or larger, then there exists a hamiltonian path in G.
Proof. (1) G is connected:
Suppose G has two or more disconnected components. Let v1 be a vertex in
one component that has n1 vertices and v2 be a vertex in another component
that has n2 vertices.
Since the degree of v1 is at most n1 - 1 and the degree of v2 is at most n2 -1,
the sum of their degrees is at most n1 + n2 - 2 < n - 1, contradicts to the
assumption.

(2) Construct a hamiltonian path:


Let there be a length p-1 (p < n) path, (v1, v2, v3, , vp). Both v1 and vp are
adjacent only to the vertices that are in the path.
There is a cycle containing exactly the vertices v1, v2, v3, , vp.
Assume v1 is adjacent to v i

,v i 2 , ...,v i k , where 1 < ij < p.

If vp is adjacent to one of v i 1 ,v i 1 ,
1
2

...,v i k 1 , then we have

the cycle.
If vp is not adjacent to any one of v i 1 ,v i 1 ,
1
2

...,v i k 1 , then

vp is adjacent to at most p-k-1 vertices. Contradicts to the assumption.


Pick a vertex vx that is not in the cycle. Because G is connected, there is a
vertex vk that is not in the cycle with an edge between vx and vk for some vk in
{v1, v2, v3, , vp}.
We now have the path (vx, vk, vk+1, , vj-1, vp, vp-1, ,vj, v1, v2, v3, , vk-1),
which contains p edges.
Repeat the foregoing construction until we have a path with n - 1 edges.

Theorem : There is always a hamiltonian path in a directed complete graph.


Proof. Let there be a length p-1 (p < n) path, (v1, v2, v3, , vp). Let vx be a vertex
that is not included in this path, and there is no edge from vx to v1. However, (v1,
vx) G.
Suppose that (vx, v2) is also an edge in the path. Replace the edge (v1, v2) in
the original path with the two edges (v1, vx) and (vx, v2) so that the vertex vx
will be included in the argument path.
If there is no edge from vx to v2, then there must be an edge (v2, vx) in the path
and we can repeat the argument.
If we find that it is not possible to include vertex vk in any augment path by
replacing an edge (vk, vk+1) in the original path with two edges (vk, vx) and (vx,
vk+1)with 1 k p-1, then we conclude that there must be an edge (vp, vx) in the
graph.
We can repeat the argument until all vertices in the graph are included in the
argumented path.
There is no general method of solution to the problem of proving the non-existence
of a hamiltonian path or circuit in a graph.
Planar Graphs

planar graph : A graph is said to be planar if it can be drawn on a plane is such a


way that no edges cross one another, except, of course, at common vertices.
Region : A region of a planar graph is defined to be an area of the plane that is
bounded be edges and is not further divided into subareas. A region is said to be
finite if this area is finite, and is said to be infinite if its area is infinite. Clearly, a
planar graph has exactly one infinite region.

Theorem : For a connected planar graph,v - e + r = 2 (Eulers formula)


where v, e, and r are the number of vertices, edges, and regions of the graph,
respectively.

Application of Eulers formula : In any connected planar graph that has no loops
and has two or more edges,e 3v -6.

Theorem (Kuratowski): A graph is planar if and only if it does not contain any
subgraph that is isometric to o either K5 or K3,3.

Tree: A part of a graph that is connected and contains no cycles.


Theorem: A connected graph possesses a tree iff there is exactly one path in
between every pair of vertices.
Theorem: A tree with n vertices has exactly n 1 vertices.
Spanning Tree: A tree containing all the vertices with exactly n 1 edges.

There are two algorithms namely Kruskals and Prim algorithms to find the MST.

Unit III

GROUP THEORY
OBJECTIVES:
After going through this unit, you will be able to know:

Binary Operation

Definition of Group, semi group, Monoid

Permutation groups

Cosets and Lagrange's theorem

Homomorphism, Isomorphism and Automorphism of Groups

Rings, integral domains and field.

INTRODUCTION:
In this chapter, we will study, binary operation as a function, and two more algebraic
structures, semigroups and groups. They are called an algebraic structure because the
operations on the set define a structure on the elements of that set. We also define
the notion of a hornomorphism and product and quotients of groups and semigroup.

BINARY OPERATION
A binary operation on a set A is an everywhere defined function f : A A A , generally
the operation is denoted by * on A, then a b A a, b A.
Properties of binary operation : Let
be a binary operation on a set A,
Then satisfies the following
properties, namely

Closure property

Associative property

Identity Property

Inverse property

Commutative property etc.

SEMIGROUP
A non-empty set S together with a binary operation is called as a semigroup if
i)
binary operation is closed
ii)
binary operation is associative
we denote the semigroup by (S, )
Commutative Semigroup :- A semigroup (S, ) is said to be
commutative if is commutative i.e. a b b aa S

Examples :

1)
2)
3)

(z, +) is a commutative semigroup


The set P(S), where S is a set, together with
operation of union is a commutative semigroup.
(Z, ) is not a semigroup
The operation subtraction is not associative

IDENTITY ELEMENT :
An element e of a semigroup (S, ) is called an identity element if e a a e a a S
Monoid A non-empty set M together with a binary operation *defined on
it, is called as a monoid if
i)
binary operation is closed
ii)
binary operation is associative and
iii)
(M, ) has an identity.
i.e. A semi group that has an identity is a monoid.
A a non-empty set G together with a binary operation defined on it is
called a group if
(i)
binary operation is close,
(ii)
binary operation is associative,
(iii) (G, ) has an identity,
(iv)
every element in G has inverse in G,
We denote the group by (G, )

Commutative (Abelian Group : A group (G, ) is said to be


commutative if is commutative. i.e. a*bb*aa, bG .
Cyclic Group : If every element of a group can be expressed as the power
of an element of the group, then that group is called as cyclic group.

The element is called as generator of the group.


If G is a group and a is its generator then we write
For example consider

G a

G {1, 1, i, i} . G is a group under the binary

operation

of

multiplication.

Note

that G i .

Because

a i,i2 ,i3 ,i 4 i, 1, i,1


SUBSEMI GROUP :

Let (S, ) be a semigroup and let T be a subset of S. If T is closed under operation ,


then (T, ) is called a subsemigroup of (S, ).
Submonoid : Let (S, ) be a monoid with identity e, and let T be a non- empty subset
of S. If T is closed under the operation and e T, then (T, ) is called a
submonoid of (S, ).
Subgroup : Let (G, ) be a group. A subset H of G is called as subgroup of G if (H, )
itself is a group.
Necessary and Sufficient Condition for subgroup : Let (G; ) be a group. A
subset H of G is a subgroup of G if and only if a, b H a b 1 H
PERMUTATION GROUP

Definition : A permutation on n symbols is a bijective function of the set

A = 1, 2,...n onto itself. The set of all permutations on n symbols is


denoted by Sn. If is a permutation on n symbols, then is completely
determined by its values 1 , 2 ..... n . We use following notation
123n
to denote
.

12345
For example
denotes the permutation on the 5 symbols
53124
(1,2,3,4,5). maps 1 to 5, 2 to 3, 3 to 1, 4 to 2 and 5 to 4.
Product of permutation : - Let A = {1,2,3,4}
1234
1234
Let
and
.
3241
432

1234 1234 1234


Then O

=

3241 432 2314


Cycle - an element sn is called a cycle of lingth r if r symbols

i1,i2....in i1 i2, i2 i3 ... in i1 .


Example : Consider following permutation
123456

. It can be expressed as a product of cycles 234165


1234 56

1234 56

2341 65

i)

Transposition :
A cycle of length two is called transposition.
For example following permutation can be expressed as a product
of transpositions.
1837 25 46
18 13 17 25 46

Even (odd) Permutation Let A {1, 2, .n). A permutation sn is even or odd according
to whether it can be expressed as the product of an even number of
transpositions or the product of an odd number of transpositions
respectively.
For example we can consider following permutation :
145 23
14 15 23
= odd no. of transpositions so is odd permutation
Example 1 : Show that defined as x y x is a binary operation on the
set of positive integers. Show that is not commutative but is associative.
Solution : Consider two positive integers x and y. By definition x y x
which is a positive integer. Hence is a binary operation.
For commutativity : x y x and y x x . Hence x y y x in general
is not commutative.
But

x ( y z) x y x

and
x ( y z) ( x y) z . is associative

( x y) z x z x .

Hence

Example 2 : Let I be the set of integers and Zm be the set of equivalence


classes generated by the equivalence relation congruent modulo m for
any positive integer m.

a)
b)
c)

Write the sets Z3 and Z6


Show that the algebraic systems (Zm, + m) and (Zm, m) are
monoids.
Find the inverses of elements in Z3 and Z4 with respect to +3 and 4
respectively.

Solution :

a)

Z3 for (Z3,+ 3) ={[0], [1], [2]}


Z6 for (Z6, + 6) = {[0], [1], [2], [3], [4], [5] }
Z3 for (Z3, 3) ={[0], [1], [2]}
Z6 for (Z6, 6) = {[0], [1], [2], [3], [4], [5] }

Example 3 : Determine whether the following set together with the binary
operation is a semigroup, a monoid or neither. If it is a monoid, specify the
identity. If it is a semigroup or a monoid determine whether it is
commutative.
i)

ii)

A = set of all positive integers.


a b max{a, b} i.e. bigger of a and
b
Set S = {1, 2, 3, 6, 12} where a b G.C.D.(a, b)

iii)

Set S ={1,2,3,6,9,18) where a b L.C.M . a,b

iv)

Z, the set of integers, where a b a b ab

v)

The set of even integers E, where a b

ab
2

vi)
Set of real numbers with a b a b 2
vii)
The set of all mn matrices under the operation
of addition.
Solution :
i) A = set of all positive integers. a b max{a, b} i.e. bigger of a and b.
Closure Property: Since Max {a, b} is either a or b a b A . Hence
closure property is verified.
Associative Property :
Since a (b c) max{{a, b}, c} max {a, b, c}
= Max{a,{b, c} } = (a.b).c
is associative.
(A, ) is a semigroup.
Existence of identity : 1 A is the identity because
1.a = Max{ 1,a}= a
aA
(A, ) is a monoid.
Commutative property : Since Max{a, b) = max{b, a) we have
a b b a Hence is commutative.
Therefore A is commutative monoid.

ii)

Set S = { 1,2,3,6,12} where a b G.C.D. (a, b)


*

1 2 3 6 12

1
2

1 1 1 1
1 2 1 2

1
2

1 1 3 3

1 2 3 6

12 1 2 3 6 12
Closure Property : Since all the elements of the table S, closure
property is satisfied.
Associative Property :Since
a (b c) a (b c) a GCD{b, c} GCD {a, b, c}

And (a b) c GCD{a, b} c GCD{a, b, c}


a (b c) (a b) c
is associative.
(S, ) is a semigroup.
Existence of identity: From the table we observe that 12 S is the
identity
(S, ) is a monoid.
Commutative property : Since GCD{a,b}= GCD{b,a) we have
a b b a . Hence is commutative.
Therefore A is commutative monoid
(iii) Set S ={ 1,2,3,6,9, 18} where a b =L.C.M. (a,b)
*

1
2

1
2

2
2

3
6

6
6

18

18

18

9 18
18 18
9

18

18 18
9

18

18 18 18 18 18 18 18
Closure Property : Since all the elements of the table S, closure
property is satisfied.
Associative Property : Since a (b c) a LCM {b, c} LCM {a, b, c}
And (a b) c LCM {a, b} c LCM {a, b, c}

a (b c) (a b) c

is associative.

(S, ) is a semigroup.

Existence of identity : From the table we observe that 1 S is the


identity.

(S, ) is a monoid.
Commutative property : Since LCM{a, b} = LCM{b, a} we have
a b b a . Hence is commutative.

Therefore A is commutative monoid.


(iv)

Z, the set of integers where - a * b = a + b - ab

Closure Property : - a,bz then a babza,b


so * is closure.
Associate Property : Consider a,bz

a*b *c abab *c
ababc a b ab c
ab abc acbc abc
ab c ab ac bc abc
a* b * c a* b c bc
a b c bc a b c bc
a b c bc ab ac abc
(2)From 1 & 2

a*b * c a* b * c a,b,c z
* is associative
(z, &) is a semigroup.
Existence of Identity : Let e be the identity element a * e = q
a + e - q.e = a
a + e - a.e = a
e ( 1-a) = 0
e = 0 or a = 1
But a1
E=0
OZ is the identity element.
(Z, *) is monoid.
Commutative property : a,bz
a * b = a + b - ab
= b + a - ba
=b*a
* is commutative
(Z, *) is commutative monoid.
OZ is the identity

(1)

v)

E = set of even integers. a b

ab
2

Closure Property : Since


ab
is even for a and b even. a b E . Hence
2

closure property is verified.

abc ab
Property : Since a (b c) q bc

c (a b) c
2
4
2

is associative.

(E, ) is a semigroup.

Existence of identity : 2 E is the identity because 2 a

2a
= a a E
2

(E, ) is a monoid.
Commutative property : Since

ab ba

, we have a b b a Hence is
2
2

commutative.
(E,*) is commutative monoid.
(vi)
(vii)

-2A is identity
0 0
0 0 M is the identity

Example 4 : State and prove right or left cancellation property for a


group.
Solution : Let (G, ) be a group.
(i)
To prove the right cancellation law i.e. a b c b a c
Let a, b, cG. Since G is a group, every element has inverse in G.
b1 G
Consider a b c b
Multiply both sides by b1 from the right.
:.

(a b) b 1 (c b) b 1

a (b b 1 ) c (b b 1 )

Associative property

e a ec

b b 1 e G

a=c

eG is the identity

(ii)
To prove the left cancellation law i.e. a b c b a c
Let a, b, cG: Since G is a group, every element has inverse in G.
a1 G
Consider
a b a c
Multiply both sides by a1 from the left

a 1 (a b) a 1 (a c)

(a 1 a) b (a 1 a) c

Associative property

eb ec

a 1 a e G

b=c

eG is the identity

Example 5 : Prove the following results for a group G.


(i)

The identity element is unique.

(ii)

Each a in G has unique inverse a1

(iii)

(ab) 1 = b1a1

Solution : (i) Let G be a group. Let e1 and e2 be two identity elements


of G.
If e1 is identity element then e1e2 = e2e1 = e2 (1)
If e2 is identity element then e1e2 = e2e1 = e1 (2)

From (1) and (2) we get e1 = e2 i.e. identity element is unique.

(ii)

Let G be a group. Let b and c be two inverses of aG.


lf b is an inverse of a then ab = ba = e(1)
If c is an inverse of a then ac = ca = e(2)
Where e G be the identity element.

From (1) and (2) we get ab = ac and ba = ca.

b=c by cancellation law : i.e. inverse of aG is unique.

inverse of a G is unique.

(iii)

Let G be a group. Let a, b G.


Consider (ab)(b1a1)
=

a(bb1)a1

(ae)a1 bb1 = e, eG is identity

(ae)a1 Associative property

aa1 e = a

eaa1 = e

Associative property

Similarly we can prove (b1a1)(ab) = e.


Hence (ab) 1 = b1 a1
Example 6 : Let G be a group with identity e. Show that if a 2 e for all a in G,
then every element is its own inverse
Solution :

Let G be a group.
Given a2 = e for all aG. Multiply by a1
we get a1a2 = a1 e

a = a1
i.e. every element is its own inverse

Example 7 : Show that if every element in a group is its own inverse, then the group must
be abelian.
OR
Let G be a group with identity e. Show that if a2 = e for all a in G, then G
is abelian.
Solution :

Let G be a group.

For aG, a1G

Consider (ab) 1

(ab) 1=b1a1 reversal law of inverse.

ab=ba every element is its own inverse

G is abelian.

Example 8 :Let Zn denote the set of integers (0, 1, .. , n-1). Let be binary operation
on Zn such that ab = the remainder of ab divided by n.
i)
Construct the table for the operation for n=4.
ii)
Show that (Zn, ) is a semi-group for any n.
iii)
Is (Zn, ) a group for any n? Justify your answer.
Solution : (i) Table for the operation for n = 4.
0 1 2 3

(ii)

0
1

0 0 0 0
0 1 2 3

0 2 0 2

0 3 2 1

To show that (Zn, ) is a semi-group for any n.

Closure property : Since all the element in the table


{0, 1, , n-1}, closure property is satisfied.

Assiciative property : Since multiplication modulo n is associative,


associative property is satisfied.

(Zn, ) is a semi-group
(iii) (Zn, ) is not a group for any n.
Example 9 : Consider the group G = {1,2,3,4,5,6} under multiplication
modulo 7.
(i)
Find the multiplication table of G
(ii)
Find 21, 31, 61.
(iii) Find the order of the subgroups generated by 2 and 3.
(iv)
Is G cyclic?
Solution : (i) Multiplication table of G
Binary operation is multiplication modulo 7.
* 1 2 3 4 5 6
1 1 2 3 4 5 6
2 2 4 6 1 3 5
3 3 6 2 5 1 4
4 4 1 5 2 6 3
5 5 3 1 6 4 2
6 6 5 4 3 2 1
From the table we observe that 1G is identity.
(ii)

To find 21, 31, 61.


From the table we get 21 = 4, 31 = 5, 61 = 6

iii)

To find the order of the subgroups generated by 2.


Consider 2 = 1 = Identity, 21 = 2; 22 = 4, 23 = 1 = Identity
< 2 > = {21, 22, 23}
Order of the subgroup generated by 2 =3
To find the order of the subgroups generated by 3.
Consider 3 = 1 = identity, 31 = 3, 32 = 2, 33 = 6, 34 = 4, 35 = 5, 36 =
1 = Identity
< 3 > = {31, 32, 33,34, 35, 36}
Order of the subgroup generated by 3 = 6

(iv)

G is cyclic because G = < 3 >.

Example 10 : Let G be an abelian group with identity e and let H = {x/x2


= e). Show that H is a subgroup of G.
Solution : Let x, yH x2 = e and y2 = e
x1 = x and y1 = y
Since G is abelian we have xy = yx xy1 = yx
Now (xy1)2 =
(xy1)(xy1) = (xy1)(y1x)
=
(xy1)(yx) = x(y1y)x
=
x(e)x
=
x2 = e

xy1 H

H is a subgroup.
Example 16 : Let G be a group and let H = (x/xG and xy = yx for all
yG}. Prove that H is a subgroup of G.
Solution : Let x, z H xy = yx for every yG x = yxy1.
Similarly zy = yz for every yG
z = yzy1.
Now consider xz1
=
(yxy1)(yzy1) 1
yxy 1yz 1y 1yxz 1y 1

(x.z1)y = y(xz1) H.
xz1 H
H is a subgroup

Example 17 : Find all subgroups of (Z,) where is the operation


addition modulo 5. Justify your answer.
Solution:
0 1 2 3 4
0
1

0 1 2 3 4
1 2 3 4 0

2 3 4 0 1

3 4 0 1 2

4 0 1 2 3

Example 18 : Let G be a group of integers under the operation of


addition. Which of the following subsets of G are subgroups of G?
(a)
the set of all even integers,
(b)

the set of all odd integers. Justify your answer.

Solution:
a)
Let H= set of all even integers.
We know, additive inverse of an even number is even and sum of
two even integers is also even. Thus for a,bH we have ab1H.
Hence H is a subgroup of G.
b)

Let K = set of all odd integers.


We know, additive inverse of an odd number is odd and sum of
two odd integers is even.
Thus for a,bK we have ab1K.
Hence K is not a subgroup of G.

Example 19 : Let (G, ) be a group and H be a non-empty subset of G.


Show that (H, ) is a subgroup if for any a and b in H, ab1 is also in H.

Solution :
(i)

Let a, a H

(ii)

(iii)

i.e. e H

The identity element H.

Let e, a H

a a1 H.
ea1 H.

i.e. a1 H

Every element has inverse H.

Let a, b H. b1 H.

a(b1) 1 H. i.e. ab H.

Closure property is satisfied.


(iv)

Every element in H is also in G. And G is a group. So associative


property is satisfied by the elements of H. Hence associative
property is satisfied by the elements of H.
Hence H is a group. But H is a subset of G. H is a subgroup
of G.

Example 20 : Let H and K be subgroups of a group G. Prove that HK is


a subgroup of G.
Solution : If H is a subgroups of a group G, then for any a, b H,
ab1 H.
Similarly, if K is a subgroups of a group G, then for any a, b K,
ab1 K.
Now if a, b HK, a, b H and a, b K. ab1 H and ab1 K.
Hence ab1 HK.

HK is a subgroup of G.

HOMOMORPHISM, ISOMORPHISM AND


AUTOMORPHISM OF SEMIGROUPS

Homomorphism : Let (S, ) and (T, ) be two semigroups. An


everywhere defined function
f : ST is called a homomorphism from (S, ) to (T, ) if
f(a b) = f(a) f(b) a, b S
Isomorphism : Let (S, ) and (T, ) be two semigoups. A function
f : S T is called a isomorphism from (S, ) to (T, ) if
(i)
it is one-to-one correspondence from S to T (ii)
f(a b) = f (a)
f (b) a, b S
(S, ) and (T, ) are isomorphic is denoted by ST .
Automorphism : An isomorphism from a semigroup to itself is called an
automorphism of the semigoup. An isonorptism f:ss I is
automorphism.

called

HOMOMORPHISM, LSOMORPHISM AND AUTOMORNHISM OF MONOIDS :

Homomorphism : Let (M, ) and (M, ) be two monoids. An


everywhere defined function f : M M is called a homomorphism from
(M, ) to (M, ) if f (a b) = f(a) f(b) a, b M
Isomorphism : Let (M, ) and (M, ) be two monoids. A function
f : M M is called a isomorphism from (M, ) to (M, ) if
(i)

it is one-to-one correspondence from M to M (ii) f is onto.

(iii)

f(a b = f (a) f (b) a, bM

(M ) and (M, ) are isomorphic is denoted by M M.


Automorphism :An isomorphism from a monoid to itself is called an automorphism
of the monoid. An isomorphism f:MM is called automorphism
of monoid.

HOMOMORPHISM, ISOMORPHISM AND AUTOMORPHISM O F GROUPS :

Homomorphism : Let (G, ) and (G, ) be two groups. An everywhere


defined function f : G G is called a homomorphism from (G, ) to (G,
) if
f (a b) = f (a) f (b) a, b G
Isomorphism : Let (G, ) and (G, ) be two groups. A function
f : GG is called a isomorphism from (G, ) to (G, ) if
(i)
it is one-to-one correspondence from G to G (ii) f is onto.
(iii) f(a b) = f (a) f (b)
a, bG
(G, ) and (G, ) are isomorphic is denoted by G G.
Automorahism: An isomorphism from a group to itself is called an
automorphism of the group. An isomorphism f:GG is called
Automorphism
Theorem : Let (S, ) and (T, ) be monoids with identity e and e,
respectively. Let f : S T be an isomorphism. Then f(e) = e.
Proof : Let b be any element of T. Since f is on to, there is an element a in
S such that f(a) = b
Then

a ae
b f (a) f (a e) f (a) f (e) b ' f (e) (f is isomorphism)

Similarly, since a e a ,

b f (a) f (e a) f (e * a) f (e) ' a

Thus for any ,bT,


b b ' f (e) f (e) ' b

which means that f(e) is an identity for T.


Thus since the identity is unique, it follows that f(e)=e
Theorem : Let f be a homomorphism from a semigroup (S, ) to a semigroup
(T, ). If S is a subsemigroup of (S, ), then F(S) = {t T | t = f (s) for some s
S},The image of S under f, is subsemigroup of (T, ).
Proof : If t1, and t2 are any elements of F(S), then there exist s1 and s2 in S with
t l= f(s1) and t2 = f(s2). Therefore,
t1 t2 f (s1 ) f (s2 ) f (s1 s2 ) f (s2 s1 ) f (s2 ) f (s1 ) t2 t1
Hence (T, ') is also commutative.

Example 1 : Let G be a group. Show that the function f : G G defined


by f(a) = a2 is a homomorphism iff G is abelian.

Solution :
Step-1 : Assume G is abelian. Prove that f : G G defined by f(a) = a2 is
a homomorphism.
Let a,bG.
f(a) = a2 , f(b) = b2 and f(ab) = (ab)2 by definition of f.

f(ab)=(ab)2
=
(ab)(ab).
=
a(ba)b
associativity
=
a(ab)b
G is abelian
=
(aa)(bb)
associativity
=
a 2 b2
=
f(a)f(b)
definition of f
f is a homomorphism.
Step 2 :
ya 2 GaGst

f(a)ya 2
f is onto.
Step-3 : Assume, f : G G defined by f(a) = a2 s a homomorphism.
Prove that G is abelian.
2
Let a,bG.
f(a) = a , f(b) = b2 and f(ab) = (ab)2 by definition of f.

f(ab) = f(a)f(b)
2
(ab) = a2 b2
(ab)(ab) = (aa)(bb)
a(ba)b = a(ab)b
ba = ab
G is abelian.

f is homomorphism
definition of f
associativity
left and right cancellation taws

Example 3 : Let G be a group and let a be a fixed element


of G. Show

that the function

f a : G G defined by

f a ( x) axa 1 for xG is an

isomorphism.
Solution :
Step-1: Show that f is 1-1.
f a ( x) axa

Consider fa(x) = fa(y)

axa1 = aya1

x=y

f is 1- 1

for x, y G
definition of f
left and right cancellation laws

Step 2 :
yaxa 1GxGs.t.
fa (x)axa 1
f is onto.
Step-3 : Show that f is homomorphism.
For x, yG
f ( x) a x a 1 ,

f ( y) a y a 1 and f ( x y) a ( x y) a 1

Consider f ( x y) a ( x y) a 1

f ( x y) a ( x e y) a 1

for

x, yG

eG is identity

= a ( x a 1 a y) a 1

a 1 a e

= (a x a 1 ) (a y a 1) associativity

f ( x y) f ( x) f ( y)

f is homomorphism.
Since f is 1-1 and homomorphism, it is isomorphism.
Example 2 : Let G be a group. Show that the function f : G G defined
by f(a) = a1 is an isomorphism if and only if G is abelian.
Solution :
Step-1: Assume G is abelian. Prove that f : G G defined by f(a) = a1 is
an isomorphism.
i)
ii)

Let f(a)=f(b)
a1 = b1
a = b

f is 1- l.

aGa G
x1 G
f x x 1

f is onto.
iii)
Let a,bG.
f(a) = a1, f(b) = b1 and f(ab) = (ab)
definition of f.
f(ab) = (ab) 1
= b1a1
reversal law of inverse
= a1b1
G is abelian
= f(a)f(b)
definition of f.

f is a homomorphism.
Since f is 1-1 and homomorphism, it is isomorphism.

by

Step 2 : Assume f : G G defined by f(a) = a1 is an isomorphism.


Prove that G is abelian.
Let a, bG
f(a) = a1, f(b) = b1 and f(ab) = (ab) 1 by definition of f
f(ab) = f(a)f(b)
f is homomorphism

1
1 1
(ab) = a b
definition of f

1 1
1 1

b a =a b
reversal law of inverse
G is abelian.
Example 3 : Define (Z, +) (5Z, +) as f(x) = 5x, where 5Z=(5n : n
Z). Verify that f is an isomorphism.
Solution:
Step -1
Consider

Show that f is 1-l.


f(x) = f(y)
for x, yG
5x = 5y
definition of f
x=y
f is 1-1

Step 2 :
5xG,x G
s.t.f(x)5x
f is onto.
Step-3: Show that f is homomorphism.
For x y G
f(x) = 5x, d(y) = 5y and f(x + y)
5(x+y)
Consider f(x+y) = 5(x+y)
for x, y G
= 5x + 5y

f(x+y) = f(x) + f(y)

f is homomorphism.
Since f is 1-1 and homomorphism, it is isomorphism.
Example 4 : Let G be a group of real numbers under addition, and let G
be the group of positive numbers under multiplication. Let f : G G be
defined by f(x) = ex. Show that f is an isomorphism from G to G
OR
Show that the group G = (R,+) is isomorphic to G = (R+, x) where R is
the set of real numbers and R+ is a set of positive real numbers.
Solution :
Step 1:Show that f is 1-1.
Consider f(x) = f(y)

ex = ey

x=y

for x,yG
definition of f
f is 1-1.

Step 2 : If xG1, then log x G and f .log x elog x x so f is onto.

Step-3 : Show that f is homomnrphism.


For x, yG
f(x) = ex, f(y) = ey and f(x+y) = e(x+y)
Consider f(x + y) = e(x + y) for x, y G
=
exey

f(x + y) = f(x) f(y) f is homomorphism.


Since f is 1-1 and homomorphasm, it is isomorphism.
Example 5 : Let G = {e, a, a2, a3, a4, a5} be a group under the operation
of aiai ar , where i + j r(mod 6). Prove that G and Z6 are isomorphic
Solution :
Step - I : Show that f is l-I.
Let x = ai, and y = aj .
Consider f(x) = f(y)

f(ai) = f(aj)

ai = aj

x=y

for x, y G
definition of f
f is 1-1.

Step-2 : Show that f is homomorphism.


Let x = a and y = a x, y G
f(ai) = i , f(aj) j and f(x + y) = f(ai aj)
Consider f(x+y) = f(aiaj) = f(a)
where i + j = r(mod 6)
=
R
=
i+j
=
f(ai) + f(aj)
f is homomorphism.
f(x y) = f(x) + f(y)

Since f is 1-1 and homomorphism, it is isomorphism.


Example 6 : Let T be set of even integers. Show that the semigroups (Z,
+) and (T, +) are isomorphic.
Solution : We show that f is one to one onto .
Define f : (Z, +) (T, +) as f(x) = 2x
1)
Show that f is l-1
Consider f(x) = f(y)
2x = 2y
x = y
f is 1-l.
2)

Show that f is onto


y = 2x x = y/2 when y is even.
for every yT there exists xZ.
f is onto.
f is isomorphic.

3)

F is homorphism
F (x + y) = 2 (x + y)
= 2x + 2y
= f(x) + f(y)
f is honomorphism.

Example 7 : For the set A = {a,b,c} give all the permutations of A. Show
that the set of all permutations of A is a group under the composition
operation.

Solution : A={a,b,c}.
a b c
f0
,
a b c
a b c
f3
,
b a c

S3= Set of all permutations of A.


a b c
a b c
f1
f2
,

c b a
a c b

a b c
a b c
f4
f5

,
b c a
c a b`

Let us prepare the composition table.


0 f 0 f1 f 2

f3

f4

f 2 f3
f 4 f5

f4

f5

f2

f3

f3

f1

f1

f2

f4

f3 f 4
f 4 f3

f0 f 4
f5 f0
f1

f2

f5

f0

f5

f5 f 2

f3

f1

f0

f4

f0

f 0 f1

f1

f1 f0
f 2 f3

f2
f3

f5

i)

Closure Property: Since all the elements in the composition table


S3, closure property is satisfied.

ii)

Associative Property: Since composition of permutations is


associative, associative property is satisfied.

iii)

Existance of Identity: From the table we find that fo is the


identity

iv)

Existance of Inverse: From the composition table it is clear that


f01 = f0, f11 = f1, f21 = f2, f31 = f3, f41 = f5, f51 = f4

Every element has inverse in S3. Hence S3 is a group.

COSET AND NORMAL SUBEROUP:


Left Coset : Let (H, ) be a subgroup of (G, ). For any a G, the set of
aH defined by aH {a h / h H } is called the left coset of H in G

determined by the element aG. The element a is called the representative


element of the left coset aH.
Right Coset : Let (H, ) be a subgroup of (G, ). For any a G, the set
of Ha defined by

Hah* a | h H
is called the right coset of H in G determined by the element aG. The
element a is called the representative element of the right coset Ha.
Theorem : Let (H, ) be a subgroup of (G, ). The set of left cosets of H
in G form a partition of G. Every element of G belongs to one and only one
left coset of H in G.
Lagrange Theorem: The order of a subgroup of a finite group divides
the order of the group.
Corollary : If (G, ) is a finite group of order n, then for any aG, we
must have an=e, where e is the identity of the group.
Normal Subgroup : A subgroup (H, ) of (G, ) is called a normal
subgroup if for any aG, aH = Ha.
Example 8 : Determine all the proper subgroups of symmetric group (S3,
o). Which of these subgroups are normal?
Solution : S = {1, 2, 3}. S3 = Set of all permutations of S.
S3 = {f0, f1, f2, f3, f4, f5 } where
1 2 3
f0
,
1 2 3

1 2 3
f1
,
1 3 2

1 2 3
f3

3 2 1

1 2 3
f3
,
2 1 3

1 2 3
f4
,
2 3 1

1 2 3
f5

3 2 1

Let us prepare the composition table.


0 f 0 f1 f 2

f3

f4

f5

f0

f0

f1

f2

f3

f4

f5

f1

f1

f0

f4

f5

f2

f3

f2

f2

f3

f0

f4

f3

f1

f3

f3

f4

f5

f0

f1

f2

f4

f4

f3

f1

f2

f5

f0

f5

f5

f2

f3

f1

f0

f4

From the table it is clear that {f0, f1}, {f0, f2,}, {f0, f3) and {f0, f4, f5} are
subgroups of (S3, 0): The left cosets of {f0, f1} are {f0, f1}, {f2, f5}, {f3, f4}.
While the right cosets of {f0, f1} are {f0, f1}, {f2, f4}, {f3, f5}. Hence {f0,
f1} is not a normal subgroup.
Similarly we can show that {f0, f2} and {f0, f1} are not normal subgroups.
On the other hand, the left and right cosets of {f0, f4, f5} are {f0, f4, f5} and
{f1, f2, f3}.
Hence {f0, f4, f5} is a nomal subgroup.
Example 9 : Let S = {1, 2, 3}. Let G = S3 be the group of all
permutations of elements of S, under the operation of composition of
permutations.
1 2 3
Let H be the subgroup formed by the two permutations
and
1 2 3
1 2 3
3 2 1 . Find the left coset of H in G. Is H a normal subgroup? Explain

your notion of composition clearly.


Solution : Let
1 2 3
f0
,
1 2 3

1 2 3
f1
,
1 3 2

1 2 3
f3

3 2 1

1 2 3
f3
,
2 1 3
H={f0, f2}

1 2 3
f4
,
2 3 1

1 2 3
f5

3 2 1

Left Cosets of H in G :
f0H = {f0f0, f0f2} = {f0, f2}
f2H = {f2f0, f2f2} = {f2, f0}
f4H = {f4f0, f4f2} = {f4, f1}

f1H = {f1f0, f1f2} = {f1, f4}


f3H = {f3f0, f3f2} = {f3, f5}
f5H = {f5f0, f5f2} = {f5, f3}

Right Cosets of H in G
Hf0 = {f0f0, f2f0} = {f0, f2}
Hf1 = {f0f1, f2f1}={f1, f3}
Since f1 H Hf1 , H is not a normal subgroup of G.
Example 10 : Define a normal sub-group. Let S3 = Group of all
permutations of 3 elements (say 1, 2, 3). For the following subgroups of S,
find all the left cosets . Subgroup of A = {1,(1,2)}
Where I = identity permutation, (1, 2) is a transposition. Is A a normal
subgroup. State a normal subgroup of the above group if it exists.
Solution :
H = {f0, f3}
The left cosets of H in G are as follow.
f0H = {f0, f3}
f1H = {f1, f5}
f3H = {f3, f0}
f4H = {f4, f2}
Consider a right coset
Hf1 = {f1, f4}
Since f1H Hf1, H is not a normal subgroup of G.

f2H = {f2, f4}


f5H = {f5, f1}

RING: An algebraic structure (R, +, o) is said to be a Ring if it satisfies :


(R, +) is a commutative Group.
(R, o) is a semigroup and
(R, +, o) satisfies the distributive property.
FIELD: An algebraic structure (F, +, o) is said to be a Field if it satisfies :
(F, +) is a commutative Group.
(F, o) is a commutative group and
(F, +, o) satisfies the distributive property.
Zero Divisor: A commutative ring is said to have a zero divisor if the product of two nonzero element is zero. For example, the product of two non- zero matrices may zero.
INTEGRAL DOMAIN: A commutative without a zero divisor is called an integral
domain.
THEOREM: Every finite integral domain is a field.
THEOREM: Every field is an integral domain.

Unit IV
LATTICE THEORY, BOOLEAN ALGEBRA AND
CODING THEORY
OBJECTIVES:

After going through this unit, you will be able to :

Define basic terminology associated with lattice theory.

Boolean lattices and Boolean algebras

Coding theory

LATTICES
BASIC TERMINOLOGY
Definition:
A poset is a lattice if every pair of elements has a lub (join) and a glb (meet).

Least upper bound (lub)

Let (A, ) be a poset and B be a subset of A.


1. An element a A is an upper bound for B iff for every element a' B, a' a.
2. An element a A is a least upper bound (lub) for B iff a is an upper bound for B and
for every upper bound a' for B, a a'.
Greatest lower bound (glb)

Let (A, ) be a poset and B be a subset of A.


1. An element a A is a lower bound for B iff for every element a' B, a a'.
2. An element a A is a greatest lower bound (glb) for B iff a is a lower bound for B
and for every lower bound a' for B, a' a.
Theorem:
Let (L, ) be a lattice, For any a, b, c L,
(i) a*a = a (i') a + a = a (idempotent)
(ii) a*b=b*a (ii') a + b = b + a (Commutative)
(iii) (a*b)*c= a*(b*c) (iii') (a + b) + c = a + (b + c) (Associative)

(iv) a*( a+ b) = a (iv') a + (a*b) = a (Absorption)


Theorem:
Let (L, ) be a lattice for any a, b L, the following
property holds.
A ba*b = a a + b = b
Theorem:
Let (L, ) be a lattice, for any a, b, c L, the following
properties hold.
B c => a*b a*c, a + b a + c
Theorem:
Let (L, ) be a lattice, For any a, b, c L, the following

properties hold.
a b ^ a c => a b + c
a b ^ a c => a b*c
b a ^ c a => b*c a
b a ^ c a => b + c a
Theorem:
Let (L, ) be a lattice, For any a, b, c L, the
following inequalities hold.
a +(b*c) (a + b)*(a + c)
(a*b )+ (a*c) a*(b + c)
BOOLEAN ALGEBRA: A complemented distributive lattice is called a Boolean Algebra.
Theorem:
Let (A, *, +) be an Boolean algebra which satisfies the

1. Idempotent law, (a*a=a, a+a=a)

2. Commutative law, (a*b=b*a, a+b=b+a)


3. Associative law, ( (a*b)*c= a*(b*c), (a + b) + c = a + (b + c)
4. Absorption law ( a*(a + b) = a, a + (a*b) = a )
Then there exists a lattice (A, ), such that * is a glb, + is a lub,
and is defined as follows:
x y iff x*y = x
x y iff x + y = y

Definitions
Algebraic system :A lattice is an algebraic system (L, *, +) with two binary operations
* and + on L which are both (1) commutative and (2) associative and (3) satisfy the
absorption law.
Sublattice : Let (L, *, +) be a lattice and let S be a subset of L. The algebra (S, *, +) is a
sublattice of (L, *, +) iff S is closed under both operations * and +.
Lattice homomorphism: Let (L, *, +) and (S, ^,V) be two lattice. A mapping g:LS is
called a lattice homomorphism from the lattice (L, *, +) to (S, ^ , V) if for any a, b L,
g(a*b) = g(a) ^ g(b) and g(a + b) = g(a) V g(b).
Order-preserving : Let (P, ) and (Q, ) be two partially ordered sets, A mapping
f: P Q is said to be order-preserving relative to the ordering in P and ' in Q iff for
any a, b P such that a b, f(a) ' f(b) in Q.
Complete Lattice: A lattice is called complete if each of its nonempty subsets has a
least upper bound and a greatest lower bound.
Greatest and Least elements
Let ( A, )> be a poset and B be a subset of A.
1. An element a B is a greatest element of B iff for every element a' B, a' a.
2. An element a B is a least element of B iff for every element a' B, a a '.
Least upper bound (lub)
Let ( A, ) be a poset and B be a subset of A.
1. An element a A is an upper bound for B iff for every element a' B, a' a.
2. An element a A is a least upper bound (lub) for B iff a is an upper bound for B and
for every upper bound a' for B, a a'.

Greatest lower bound (glb)


Let ( A, ) be a poset and B be a subset of A.
1. An element a A is a lower bound for B iff for every element a' B, a a'.
2. An element a A is a greatest lower bound (glb) for B iff a is a lower bound for B
and for every lower bound a' for B, a' a.
Maximal and Minimal Elements: Let (A, R) be a poset. Then a in A is a minimal
element if there does not exist an element b in A such that bRa. Similarly for a maximal
element.
Upper and Lower Bounds
Let S be a subset of A in the poset (A, R). If there exists an element a in A such that sRa for all s
in S, then a is called an upper bound. Similarly for lower bounds.
Bounds of the lattice :The least and the greatest elements of a lattice, if they exist, are called the
bounds of the lattice, and are denoted by 0 and 1 respectively.

Bounded lattice: In a bounded lattice (L, *, +, 0, 1), an element b L is called a


complement of an element a L, if a*b=0,
a + b =1.
Complemented lattice :A lattice (L, *, +, 0, 1) is said to be a complemented lattice if every
element of L has at least one complement.

Distributive lattice :A lattice (L, *, +) is called a distributive lattice if for any a, b, c


L, a*(b + c) = (a*b) + (a*c) + (b*c) = (a + b)*(a + c)

EXAMPLE:
Construct the Hasse diagram of (P({a, b, c}), ).
The elements of P({a, b, c}) are

{a}, {b}, {c}


{a, b}, {a, c}, {b, c}
{a, b, c}
The digraph is

In the above Hasse diagram, is a minimal element and {a, b, c} is a maximal element.
In the poset above {a, b, c} is the greatest element. is the least element.
In the poset above, {a, b, c}, is an upper bound for all other subsets. is a lower bound
for all other subsets.
{a, b, c}, {a, b} {a, c} and {a} are upper bounds and {a} is related to all of them, {a}
must be the lub. It is also the glb.
EXAMPLE:
In the poset (P(S), ), lub(A, B) = A B. What is the glb(A, B)?

Solution:
Consider the elements 1 and 3.
Upper bounds of 1 are 1, 2, 4 and 5.
Upper bounds of 3 are 3, 2, 4 and 5.
2, 4 and 5 are upper bounds for the pair 1 and 3.
There is no lub since

- 2 is not related to 4
- 4 is not related to 2
- 2 and 4 are both related to 5.
There is no glb either.
The poset is n o t a lattice.

EXAMPLE:

Determine whether the posets represented by each of the following Hasse diagrams have
a greatest element an a least element.

rete Mathematics
Solution
The least element of the poset with Hasse diagram (a) is a. This poset has no greatest
element.
The poset with Hasse diagram (b) has neither a least nor a greatest element.
The poset with Hasse diagram (c) has no least element. Its greatest element is d.
The poset with Hasse diagram (d) has least element a and greatest element d.
EXAMPLE:

Find the lower and upper bounds of the subsets {a, b, c}, {j, h}, and {a, c, d, f } and find
the greatest lower bound and the least upper bound of {b, d, g}, if they exist.

Solution

The upper bounds of {a, b, c} are e, f, j, h, and its only lower bound is a.
There are no upper bounds of {j, h}, and its lower bounds are a, b, c, d, e, f .
The upper bounds of {a, c, d, f } are f, h, j, and its lower bound is a.
The upper bounds of {b, d, g} are g and h. Since g _ h, g is the least upper bound.
The lower bounds of {b, d, g} are a and b. Since a _ b, b is the greatest lower bound.
EXAMPLE:
Determine whether the posets represented by each of the following Hasse diagrams are
lattices.

Solution

The posets represented by the Hasse diagrams in (a) and (c) are both lattices because in
each poset every pair of elements has both a least upper bound and a greatest lower
bound.
On the other hand, the poset with the Hasse diagram shown in (b) is
not a lattice, since the elements b and c have no least upper bound. To see this note that
each of the elements d, e and f is an upper bound, but none of these three elements
precedes the other two with respect to the ordering of this poset.
EXAMPLE:

Determine whether (P(S ), ) is a lattice where S is a set.


Solution
Let A and B be two subsets of S . The least upper bound and the greatest lower bound of
A and B are A U B and A B, respectively.
Hence (P(S ), ) is a lattice.

CODES AND GROUP CODES


INTRODUCTION :
In todays modern world of communication, data items are
constantly being transmitted from point to point.
Different devices are used for communication. The basic unit of
information is message. Messages can be represented by sequence of
dots and das
Let
B = {0,1}be the set of bits. Every character or symbol can be
represented by sequence of elements of B. Message are coded in Os and 1s
and then they are transmitted. These techniques make use of group theory. We
will see a brief introduction of group code in this chapter. Also we will see the
detection of error in transmitted message.
B 0,1 is a group under the binary operation whose
The set
table is as follows :

We have seen that B is a group.


It follows from theorem - If G1 and G2 are groups then
GG1G 2 is a group with binary operation defined by

a1,b1 a 2,b2 a1,a2,b1,b 2 . So

Bm BBB (m factors) is

group
under
the
operation

defined
x1,x 2 ....xm y1,y 2 ym x1y1,x 2 y 2,....xm ym
observe that B m has 2m elements. i.e. order of group Bm is 2m.
a

Important Terminology :
Let us choose an integer
e:Bm Bn .

1)

by

nm and one-to-one function

Encoding Function :
The function e is called an (m, n) encoding function. It means
that every word in Bm as a word in Bn.

2)

Code word :
If bBm then e(b) is called the code word

3)

Weight :
For xBn the number of 1s in x is called the weight of x and is
denoted by x .
e.g.

i) x10011B5 w x 3
ii) x001B 3 w x

4)

xy Let x, yBn , then xy is a sequence of length n that

has 1s in those positions x & y differ and has Os in those positions x


& y are the same. i.e. The operation + is defined as 0 + 0 = 0
0+1
=1 1+1
=0 1+0=1

e.g. if x,yB 5
x00101,y10110
xy10011
w(xy)3
5)

Hamming Distance :
Let x,yBm . The Hamming Distance x, y between x and y is

the weight of xy . It is denoted by xy . e.g. Hamming distance


between x & y can be calculated as follows : if x = 110110, y = 000101
xy = 110011 so xy = 4.
6)

Minimum distance :
Let x,yBn. then minimum distance = min

d x, y /x, yB .
n

Let x1,x 2 x n are the code words, let any xi ,i1 n is a


transmitted word and y be the corresponding received word. Then yx k
if d x k , y is the minimum distane for k = 1, 2, --- n. This criteria is
known as minimum distance criteria.
7)

Detection of errors :
Let e : Bm Bn mn is an encoding function then if minimum
distane of e is ( k + 1) then it can detect k or fewer errors.
8)

Correction of errors :
Let e : Bm Bn mn is an encoding function then if minimum
distance of e is (2k + 1) then it can correct k or fewer errors.
Weight of a code word : It is the number of 1s present in the given code
word.
Hamming distance between two code words : Let x x1x2 ...xm and
y y1 y2 ... ym be two code words. The Hamming distance between
them, x, y , is the number of occurrences such that xi yi for i 1,m .
Example 1 : Define Hamming distance. Find the Hamming distance
between the codes.
(a) x 010000, y 000101
(b) x 001100, y 010110
Solution : Hamming distance :
(a) x, y x y 010000 000101 010101 3
(b) x, y x y 001100 010110 011010 3

4,3 decoding function


y y1 y2 ... ym1 , d y y1 y2 ... ym .

Example 2 : Let d be the


d : B 4 B 3 . If

defined by

Determine d y for the word y is B4 .


(a) y 0110

(b) y 1011

Solution : (a) d y 011

(b) d y 101

Example 3 : Let d : B 6 B 2 be a decoding function defined by for


y y1 y2 ... y6 . Then d y z1z2 .
where
zi 1 if y1, yi 2 , yi 4 has at least two 1s.

0 if y1, yi 2 , yi 4 has less than two 1s.

Determine d y for the word y in B6 .


(a) y 111011
(b) y 010100
Solution : (a) d y 11

(b) d y 01

Example 4 : The following encoding function


the

parity

m,m 1 check

e b b1b2 ...bm bm1

code.

f : B m B m1 is called

If b b1 b2 ...bm B m ,

where
bm1 0 if b is even.
= 1 if b is odd.
Find e b if (a) b 01010

(b) b 01110

define

Solution : (a) e b 010100

(b) e b 011101

Example 5 : Let e : B 2 B 6 is an (2,6) encoding function defined as


e(00) = 000000,
e(01) = 011101
e(10) = 001110,
e(11) = 111111
a) Find minimum distance.
b) How many errors can e detect?
c) How many errors can e correts?
Solution : Let x 0 ,x1,x 2,x 3 B 6 where x 0 000000,x1 011101,
x 2 001110,x 3 111111
w x 0 x1 w 0111014
w x 0 x 2 w 001110 3
w x 0 x 3 w1111116

x1x 2 w 0100113
w x1x 3 w100010
w x 2 x 3 w1100013
w

Minimum distance = e = 2
d) Minimum distance = 2
An encoding function e can detect k or fewer errors if the minimum
distance is k + 1. k12k1
The function can detect 1 or fewer (i.e. 0) error.
e) e can correct k or fewer error if minimum distance is 2k + 1.
2k + 1 = 2
1
k =
2
1
1
or less than i.e. 0 errors.
e can correct
2
2

GROUP CODE :

An m,n encoding function e : B m B n is called a group code


if range of e is a subgroup of Bn. i.e. (Ran (e), ) is a group.
Since Ran (e) CBn and if (Ran (e), ) is a group then Ran(e) is a

subgroup of Bn. If an encoding function e : B m B n (n < n) is a group


code, then the minimum distance of e is the minimum weight of a nonzero
codeword.

DECODING AND ERROR CORRECTION :


Consider an m,n encoding function e : B m B n , we require an
(n,m) decoding function associate with e as d : B n B m .
The method to determine a decoding function d is called maximum
likelihood technique.
Since Bm 2m .
Let x k Bm be a codeword, k = 1, 2, ---m and the received word is y then.

Min 1k2m d x k ,y d x i ,y for same i then xi is a codeword


which is closest to y. If minimum distance is not unique then select on
priority

MAXIMUM LIKELIHOOD TECHNIQUE :


Given an m,n encoding function e : B m B n , we often need to

determine an n,m decoding function d : B n B m associated with e.


We now discuss a method, called the maximum likelihood techniques, for
determining a decoding function d for a given e.

Since Bm has 2m

elements, there are 2m code words in Bn . We first list the code words in
a fixed order.

2 m

1
2
x ,x ,...,x

If the received word is x1 , we compute x i , x1 for 1 i 2m


s
and choose the first code word, say it is x , such that

i
min x ,x1

1i2 m

x ,x
s

s
That is, x is a code word that is closest to x1 , and the first in the

s
list. If x e b , we define the maximum likelihood decoding function
d associated with e by
d xt b

Observe that d depends on the particular order in which the code

are listed.

words in e B n

If the code words are listed in a different

order, we may obtain, a different likelihood decoding function d


associated with e.
Theorem : Suppose that e is an m,n

encoding function and d is a

maximum likelihood decoding function associated with e. Then e,d


can correct k or fewer errors if and only if the minimum distance of e is at
least 2k 1 .

Example : Let m 2,n 5

0
and H 1

0
0

1 0

1 1
0 0 .

1 0
0 1

Determine the

group code e H : B2 B5 .
Solution : We have B2 00,01,10,11 . Then e 00 00x1x 2 x 3
where

x1 0.1 0.0 0
x2 0.1 0.1 0
x3 0.0 0.1 0

e 00 00000
Now,

e 01 01x1x2 x3

where
x1 0.1 1.0 0
x2 0.1 1.1 1
x3 0.0 1.1 1

e 01 01011
Next

e 10 10 x1x2 x3
x1 1.1 0.0 1
x2 1.1 1.0 1
x3 1.0 0.1 0

e 10 10110
e 11 11101

Example : Let

0 0

1 1

1 1
be a parity check matrix. determine
0 0
1 0

0 1

0
1
H
1
0

the 3,6 group code e H : B3 B6 .


Solution : First find e 000 ,e 001 ,e 010 ,e 011 ,e 100 ,e 101 ,
e 110 ,e 111 .
e 000 000000

e 100 100100

e 001 001111

e 101 101011

e 010 010011

e 110 110111

e 100 011100

e 111 111000

e : B2 B5 such that
Example : Consider the group code defined by
e 00 00000e 01 01110e 10 10101e 11 11011 .
Decode the following words relative to maximum likelihood decoding
function.
(a) 11110
(b) 10011
(c) 10100
Solution : (a) xt 1110
Compute


x , x 01110 11110 10000 1
x , x 10101 11110 01011 3
x , x 11011 11110 00101 2


min x , x 1 x , x
1
x , xt 00000 11110 11110 4
2

e 01 01110 is the code word closest to xt 11110 .


The maximum likelihood decoding function d associated with e is
defined by d xt 01.

(b) xt 10011
Compute


x , x 01110 10011 00110 2
x , x 10101 11110 01011 3
x , x 11011 10011 01000 1
min x , x 1 x , x
1
x , xt 00000 10011 11101 4
2

e 11 11011 is the code word closest to xt 10011 .


The maximum likelihood decoding function d associated with e is
defined by d xt 11.
(c) xt 10100
Compute


x , x 01110 10100 11010 3
x , x 10101 10100 00001 1
x , x 11011 10100 01111 4


min x , x 1 x , x
1
x , xt 00000 10100 10100 2
2

e 10 10101 is the code word closest to xt 10100 .


The maximum likelihood decoding function d associated with e is
defined by d xt 10 .
0 1 1

0
1
1

Example : Let H 1
0 0 be a parity check matrix. decode the

0 1 0
0 0 1
following words relative to a maximum likelihood decoding function
associated with e H : (i) 10100, (ii) 01101, (iii) 11011.
Solution : The code words are e 00 00000,e 01 00101,e 10 10011,
e 11 11110 . Then N 00000,00101,10011,11110 . We implement
the decoding procedure as follows. Determine all left cosets of N in B5,

as rows of a table. For each row 1, locate the coset leader i , and rewrite
the row in the order.
1,i

Example : Consider the 2,4


many errors will e detect?

encoding function e as follows. How

e 00 0000,e 01 0110,e 10 1011,e 11 1100


Solution :

0000

0110

1011

1100

0000

---

0110

1011

1100

---

1101

1010

---

0111

0110
1011
1100

---

Minimum distance between distinct pairs of e 2 k 1 2 k 1.


the encoding function e can detect 1 or fewer errors.
Example : Define group code. Show that 2,5

encoding function

e : B2 B5 defined by e 00 0000,e 10 10101,e 11 11011 is a


group code.

Solution : Group Code

00000

01110

10101

11011

00000

00000

01110

10101

11011

01110

01110

00000

11011

10101

10101

10101

11011

00000

01110

11011

11011

10101

01110

00000

Since closure property is satisfied, it is a group code.


Example : Define group code. show that 2,5

encoding function

e : B2 B5 defined by e 00 00000,e 01 01110,e 10 10101 ,

e 11 11011 is a group code. Consider this group code and decode the
following words relative to maximum likelihood decoding function.
(a) 11110
(b) 10011.
Solution : Group Code

00000

01110

10101

11011

00000

00000

01110

10101

11011

01110

01110

00000

11011

10101

10101

10101

11011

00000

01110

11011

11011

10101

01110

00000

Since closure property is satisfied, it is a group code.


1
2
3
4
Now, let x 00000,x 01110,x 10101,x 11011 .

(a) xt 11110


x , x x x 01110 1110 10000 1
x , x x x 10101 1110 01011 3
x , x x x 11011 1110 00101 2

1
1
x , xt x xt 00000 11110 11110 4
2

Maximum likelihood decoding function d x t 01 .


(b) xt 10011


x , x x x 01110 10011 11101 4
x , x x x 10101 10011 00110 2
x , x x x 11011 10011 01000 1
1
1
x , xt x xt 00000 10011 10011 3
2

Maximum likelihood decoding function d xt 11.

Example : Let

0
1
H
1
0

0 0

1 1
1 1
be a parity check matrix. Determine
0 0
1 0

0 1

the 3,6 group code e H : B3 B6 .


Solution : B3 000,001,010,011,100,101,110,111
e H 000 000000e H 001 001111e H 010 010011

e H 011 011100e H 100 100100eH 101 101011


e H 110 110111eH 111 111000
Required group code = 000000 ,001111,010011,011100,100100,

101011,110111,111000

Example : Show that 2,5

encoding function e : B2 B5 defined


by e 00 00000,e 01 01110,e 10 10101,e 11 11011 is a
group code.
Test whether the following 2,5 encoding function is a group code.
e 00 00000,e 01 01110,e 10 10101,e 11 11011
Solution :

00000

01110

10101

11011

00000

00000

01110

10101

11011

01110

01110

00000

11011

10101

10101

10101

11011

00000

01110

11011

11011

10101

01110

00000

Since closure property is satisfied, it is a group code.


Example

: Show that the

3,7

encoding function e : B3 B7

defined by
e 000 0000000e 001 0010110e 010 0101000

e 011 0111110e 100 1000101e 101 1010011


e 110 1101101e 111 1111011 is a group code.
Solution :

0000000

0010110

0101000

0111110

1000101

1010011

1101101

1111011

0000000

0000000

0010110

0101000

0111110

1000101

1010011

1101101

1111011

0010110

0010110

0000000

0111110

0101000

1010011

1000101

1111011

1101101

0101000

0101000

0111110

0000000

0010110

1101101

1111011

1000101

1010011

0111110

0111110

0101000

0010110

0000000

1111011

1101101

1010011

1000101

1000101

1000101

1010011

1101101

1111011

0000000

0010110

0101000

0111110

1010011

1010011

1000101

1111011

1101101

0010110

0000000

0111110

0101100

1101101

1101101

1111011

1000101

1010011

0101000

0111110

0000000

0010110

1111011

1111011

0000000

Since closure property is satisfied, it is a group code.


Example: Consider the

3,8

encoding function e : B3 B8

defined by
e 000 0000000e 100 10100100e 001 10111000
e 101 10001001e 010 00101101e 110 00011100
e 011 10010101e 111 00110001 .
How many errors will e detect?

Solution :

00000000 10100100 10111000 10001001 00101101 00011100 10010101 00110001

0000000

00000000 10100100 10111000 10001001 00101101 00011100 10010101 00110001

10100100 10100100 00000000 00011100 00101101 10001001 10111000 00110001 10010101


10111000 00000000 00011100 00000000 001100001 10010101 10100100 00101101 10001001
10001001 10001001 00101101 00110001 00000000 10100100 10010101 00011100 10111000
00101101 00101101 10001001 10010101 10100100 00000000 00110001 10111000 00011100
00011100 00011100 10111000 10100100 10010101 00110001 00000000 10001001 00101101
10010101 10010101 00110001 00101101 00011100 10111000 10001001 00000000 10100100
00110001 00110001 10010101 10001001 10111000 00011100 00101101 10100100

0000000

Minimum distance between pairs of e 3 .


k 1 3 k 2The encoding function e can detect 2 or fewer
errors.

Example: Consider parity


check matrix H
given
by
1 1 0

0 1 1
H 1 0 0 . Determine the group code e H : B2 B5 . Decode the

0 1 0
0 0 1
following words relative to a maximum likelihood decoding function
associated with e H : 01110,11101,00001,11000 .
[Apr-04, May-07]
Solution : B2 00,01,10,11

eH 00 00x1x2 x3 where x1 0.1 0.0 0


x2 0.1 0.1 0
x3 0.0 0.1 0

eH 00 00000

eH 01 01x1x2 x3 where x1 0.1 1.0 0


x2 0.1 1.1 1
x3 0.0 1.1 1
eH 10 10x1x2 x3

where x1 1.1 0.0 1


x2 1.1 0.1 1
x3 1.0 0.1 0

eH 11 11x1x2 x3

eH 01 10110

where x1 1.1 1.0 1


x2 1.1 1.1 0
x3 1.0 1.1 1

eH 01 01011

eH 01 11101

Desired group code = 00000,01011,10110,11101

(1) xt 01110


x , x x x 01011 01110 00101 2
x , x x x 10110 01110 11000 2
x , x x x 11101 01110 10011 3
1
1
x , xt x xt 00000 01110 01110 3
2

Maximum likelihood decoding function d x t 01

(2) xt 11101


x , x x x 01110 11101 10110 3
x , x x x 10101 11101 01011 3
x , x x x 11011 11101 00000 0
1
1
x , xt x xt 00000 11101 11101 4
2

Maximum likelihood decoding function d x t 11


(3) xt 00001


x , x x x 01011 00001 01010 2
x , x x x 10110 00001 10111 4
x , x x x 11101 00001 11100 3
1
1
x , xt x xt 00000 00001 00001 1
2

Maximum likelihood decoding function d x t 00


(2) xt 11000


x , x x x 01110 11000 10011 3
x , x x x 10101 11000 01101 3
x , x x x 11011 11000 10000 1
1
1
x , xt x xt 00000 11000 11000 2
2

Maximum likelihood decoding function d xt 11

----- *** ------

Вам также может понравиться